Dalton Quizzes Flashcards

1
Q

401K Plan distributions are subject to:

A
  1. ) Distributions from 401(k) plans are subject to qualified plan distribution rules.
  2. ) 401(k) plans often allow participants to make in-service withdrawals (with-drawals before termination of employment).
  3. ) 401(k) accounts based on elective deferrals cannot be distributed before occur-rence of one of the following events: a.) Retirement b.) Death c.) Disability d.) Separation from service with the employer e.) Attainment of age 591⁄2 by the participant f.) Plan termination g.) Hardship
  4. ) Many preretirement distributions will not only be taxable but may also be sub-ject to the 10% early withdrawal penalty tax.
How well did you know this?
1
Not at all
2
3
4
5
Perfectly
2
Q

401k : A hardship withdrawal must meet the following tests.

A
  1. ) Financial needs test: The hardship must be due to an immediate and heavy financial need of the participant-employee.
  2. ) Resources test: The participant must not have other financial sources sufficient to satisfy the need.
  3. ) In addition to meeting both of these tests, the money may only be withdrawn for the following reasons:
    a. ) Payment of unreimbursed medical expenses for employee, spouse, or dependents
    b. ) Purchase of a primary residence or repair of casualty loss damages of a pri-mary residence
    c. ) Payment for up to the next 12 months of higher education expenses for the participant, the participant’s spouse, or dependent children
    d. ) Payment necessary to prevent foreclosure on the participant’s primary residence
    e. ) Burial or funeral expenses for an employee’s deceased parents, spouse, children, and dependents
    f. ) Safe harbor: Plan may identify a distribution to satisfy a financial need without the requirement that all other employee resources be exhausted if certain requirements are met.
  4. ) When a hardship withdrawal is taken, the participant’s right to make elective deferrals must be suspended. 5.) Finally, if a hardship withdrawal is approved and made, the distribution is tax-able, and a possible 10% penalty applies.
How well did you know this?
1
Not at all
2
3
4
5
Perfectly
3
Q

What’s the basis of transferred property from a divorce?

A

Transfers between divorcing spouses ALWAYS transfer their basis and their holding period, regardless of FMV at date of transfer.

How well did you know this?
1
Not at all
2
3
4
5
Perfectly
4
Q

secular trust?

A

secular trust calls for an irrevocable contribution from the employer to finance promises under a nonqualified plan, and funds held within the trust cannot be reached by the employer’s creditors.

How well did you know this?
1
Not at all
2
3
4
5
Perfectly
5
Q

According to ERISA, what are required to be distributed annually to defined benefit plan participants or beneficiaries?

A

Individual Benefit Statements are not required annually for defined benefit plans. They are however, required at least once every three years. Alternatively, defined benefit plans can satisfy this requierment if at least once each year the administrator provides notice of the availability of the pension benefit statement and the ways to obtain such statement. In addition, the plan administrator of a defined benefit plan must furnish a benefit statement to a participant or beneficiary upon written request, limited to one request during any 12-month period. There are no individual accounts in a defined benefit plan, so a specific listing of invested assets is not required.

How well did you know this?
1
Not at all
2
3
4
5
Perfectly
6
Q

Generally, younger entrants are favored in which of the following plans?

A

Cash Balance and Money Purchase Pension Plans favor younger entrants. Defined Benefit and Target Benefit Pension Plans favor older age entrants with less time to accumulate, and therefore, require higher funding levels.

How well did you know this?
1
Not at all
2
3
4
5
Perfectly
7
Q

5112Question 19 of 25Retirement and EE Benefits Quiz 5

Which of the following are common actuarial assumptions used in determining the plan contributions needed to fund the benefits of a defined benefit plan?

I. Investment performance.

II. Employee turnover rate.

III. Salary levels.

IV. Ratio of single to married participants.

A
A)I, II and III only.
Rationale
The correct answer is "A." The number of married employees is irrelevant because the benefits paid to a single employee are actuarially equivalent to benefits paid to a married couple. Investment performance has an inverse relationship to contribution levels (higher investment returns = lower contribution requirement.) Employee turnover rate affects contributions due to forfeitures. Salary scale affects funding levels because increases in the salary scale of an employee increases the required funding.
B)I and III only.
C)II and IV only.
D)IV only.
How well did you know this?
1
Not at all
2
3
4
5
Perfectly
8
Q

Which retirement plan has loan provision?

A
  1. Any type of qualified plan or 403(b) plan may permit loans; however, usually only 401(k) and 403(b) plans have loan provisions.
  2. All loans must be repaid within five years (except loans used to acquire a principal residence) and include interest. a. Loans for the purpose of acquiring a principal residence must be repaid over a reasonable period. b. The plan document will generally set forth the repayment for these loans. c. Plan loans must be available to all participants on a reasonably equivalent basis and must not be available to highly compensated employees in an amount greater than the amount made available to other employees. 1.) Plan loans are available to participants who are also self-employed owners or partners. d. Plan loans must: 1.) be adequately secured; 2.) be made in accordance with specific plan provisions; and 3.) bear a reasonable (market) rate of interest.
  3. Generally, loans are limited to one-half the present value of the participant’s nonforfeitable accrued benefit or vested account balance and cannot exceed $50,000. However, when a participant’s vested account balance is less than $20,000, exceptions to the 50% limit may be made, as illustrated as follows. a. When account balances are $10,000 or less, the vested account balance is available for loan. b. When account balances are less than $20,000, loans up to $10,000 are available. c. Loans must be amortized on a level basis with payments made by the participant-borrower at least quarterly
  4. The maximum loan amount may be further reduced by any loan balance the participant had in the one-year period preceding the loan. If it weren’t for this reduction, participants could abuse the five-year repayment rule by repaying the loan on the last possible date and then immediately taking out the loan again.
  5. Generally, loans must be repaid in full upon separation from service; if not, the outstanding balance at the time of separation is treated as a taxable distribution and possibly subject to the 10% early distribution penalty.
  6. Interest paid on plan loans secured by elective deferrals is nondeductible.
  7. Starting in 2020, retirement loans cannot be offered using a credit card or similar arrangements.
How well did you know this?
1
Not at all
2
3
4
5
Perfectly
9
Q

Which of the following is/are reason(s) employers sponsor pension plans?

I. Recruit quality employees.

II. Show stability of the company to lenders.

III. Fight/discourage collective bargaining.

IV. Provide working capital for the company.

A
A)I only.
B)I and II only.
C)I and III only.
Rationale
The correct answer is "C." Pensions do not demonstrate stability to a lender. In fact, if there is a mandatory contribution to the plan, this may affect company cash flow and credit worthiness. The sponsoring company cannot use pension assets for working capital. This would be a prohibited transaction (self-dealing).
D)I, II and IV only.
How well did you know this?
1
Not at all
2
3
4
5
Perfectly
10
Q

Corey is covered under his employer’s Profit-Sharing Plan. He currently earns $500,000 per year. The plan is top heavy. The employer made a 5% contribution on behalf of all employees. What is the company’s contribution for him?

A
A)$14,250
Rationale
The correct answer is "A." For a profit sharing plan the contribution is limited to the lesser of $57,000 (2020) or covered compensation. In this case the contribution will be limited by the covered compensation limit of $285,000. $285,000 x 5% = 14,250. The fact that the plan is top heavy is irrelevant since all employees are receiving a contribution greater than 3%.
B)$25,000
C)$57,000
D)$285,000
How well did you know this?
1
Not at all
2
3
4
5
Perfectly
11
Q

What kind of plans are belong to the employers’ contribution?

A

Employers generally contribute to Money Purchase Pension Plans, ESOPs, and Profit Sharing Plans. Employees contribute (thus contributory plans) to 401(k)s and Thrift Plans.

How well did you know this?
1
Not at all
2
3
4
5
Perfectly
12
Q

How to accurately describe a qualified group life insurance plan?

A

I. The plan must benefit 70% of all employees, or a group consisting of 85% non-key employees, or a non-discriminatory class, or meet the non-discrimination rules of Section 125.

II. Employees who can be excluded are: those with fewer than 3 years service, part-time / seasonal, non-resident aliens, or those covered under a collective bargaining unit.

III. A qualified group life insurance plan, if using a non-discriminatory classification, will have a bottom tier with benefits no less than 10% of the top tier and no more than 250% increase between tiers.

IV. The minimum group size is 10.

How well did you know this?
1
Not at all
2
3
4
5
Perfectly
13
Q

Which one favors older employees? target benefit pension plan or money purchase plan?

A

target benefit plan: the employee’s bear the investment risk and favors older employees .
An age based profit sharing plan is not a pension plan - it is a profit sharing plan.
The employer bears the risk on the cash balance plan. The money purchase plan favors younger employees.

How well did you know this?
1
Not at all
2
3
4
5
Perfectly
14
Q

Which of the conditions would prevent a deductible IRA contribution from being made by a company?

A

An active participant is an employee who has benefited under one of the following plans through a contribution or an accrued benefit during the year:

  1. qualified plan (401K and others);
  2. annuity plan;
  3. tax sheltered annuity (403(b) plan);
  4. certain government plans (does not included 457 plans);
  5. SEPs; or
  6. SIMPLEs.

Statement I is a non-qualified deferred comp plan (not one of the plans listed above) and therefore not to be taken into consideration for active participation status. Statement II & V are on the list above. For a defined benefit plan, an individual who is eligible for the plan is automatically considered an active participant. Statement “III” is not active participation, rather it is retirement, and Statement “IV” as described without contributions or forfeitures is not “active participation,” but a change in conditions regarding employer contributions or forfeitures could stem deductibility of IRA contributions.

If you are considered to be an active participant in a company plan, and your income is above certain limits, then you cannot take a tax deduction for your traditional IRA contribution

How well did you know this?
1
Not at all
2
3
4
5
Perfectly
15
Q

pertinent

A

adj. 中肯的;相关的;切题的;恰当的

How well did you know this?
1
Not at all
2
3
4
5
Perfectly
16
Q

What are the qualified plan tests for eligibility?

A

A)Ratio percentage test - Plan must cover a percentage of non-highly compensated employees that is at least 70% of the percentage of highly compensated employees covered.
B)Average benefits test - Plan must benefit a non-discriminatory employee class with benefits of at least 70% of the benefit provided highly compensated .
C)50/40 test - 50 employees or 40% of employees with a minimum of 2 out of 3 employees (unless there is only one employee in which case only 1 participant is required).
D)Plans cannot require more than 1 year of service, and an age higher than 21. The plan can require a 2-year waiting period if there is immediate 100% vesting in the plan.

How well did you know this?
1
Not at all
2
3
4
5
Perfectly
17
Q

What are the common characteristics for the traditional defined contribution plan and SEP-IRA?

A
  1. Requires a definite, written, non-discriminatory contribution allocation formula.
  2. Contributions cannot discriminate in favor of highly compensated employees.
  3. Affiliated service group rules apply.

Defined contribution plans have an employer deductibility limit of 25% of covered payroll. All defined contribution plans must have a written allocation formula so assets can be distributed in the mandated individual accounts. Employer contributions must bear uniform resemblance to compensation and cannot discriminate in favor of highly compensated. Employer contributions are not subject to any payroll related taxes. Top-heavy rules do apply to both. Both plans can integrate with Social Security (sometimes called permissible disparity). (Note: 5305-SEP does not allow permissible disparity.)

How well did you know this?
1
Not at all
2
3
4
5
Perfectly
18
Q

Permissible disparity or integration

A

允许的差异或整合

How well did you know this?
1
Not at all
2
3
4
5
Perfectly
19
Q

Substituted basis

A

Substituted basis is a more general term that can refer to either transferred basis or exchanged basis.
Substitute basis is the fair market value of an asset, reduced by gain realized, but not recognized.

How well did you know this?
1
Not at all
2
3
4
5
Perfectly
20
Q

Which of plan is required to make contributions each year?

A

It’s the pension plan.
A profit sharing plan is not required to make contribution each year.
1)Profit sharing plans fall under the broad category of defined contribution plans.
2)Profit sharing plans are best suited for companies that have unstable earnings.
3)The maximum tax deductible employer contribution to a profit sharing plan is 25% of covered compensation.

How well did you know this?
1
Not at all
2
3
4
5
Perfectly
21
Q

What is Top-Heavy Status?

A

If your plan is top heavy (meaning 60% or greater of the account balances in the plan are in key employees’ accounts) and one of these key employees has deferred into the plan, you more than likely will be required to make a 3% top-heavy contribution into your plan.

How well did you know this?
1
Not at all
2
3
4
5
Perfectly
22
Q

SELF-EMPLOYED (KEOGH) PLANS

A
  1. loans are available to owners and employees alike, if each has equal right and terms of the loans.
  2. Contributions for owners are based on net earnings rather than wages.
  3. Lump-sum distribution tax treatment allowed for employees, but not for owners, except in the case of disability.
  4. Contributions for employees must do a conversion [EE contr rate ÷ (1+ EE contr rate)] e.g., .15 ÷ (1+.15) = .13043 so owner’s contribution as a percentage of profits is lower than the employees’ percentage of wages earned.
How well did you know this?
1
Not at all
2
3
4
5
Perfectly
23
Q

What are the three most common types of qualified plans adopted by the self-employed?

A

profit-sharing, money purchase pension, and target benefit pension plans, all of which are defined contribution plans.
The three most common types of qualified plans adopted by the self-employed are

How well did you know this?
1
Not at all
2
3
4
5
Perfectly
24
Q

Section 457 plan

A
  1. Non-qualified plan
  2. Benefits taken as periodic payments are treated as ordinary income for taxation.
  3. Deferred amounts are subject to Social Security and Medicare taxes at the later of: performance of services or employee becomes vested in the benefits.
  4. Cannot exceed the smaller of $19,500 or 100% includible compensation.
  5. Section 457 plans are nonqualified, unfunded deferred compensation plans established by state and local government and tax-exempt employers.
  6. An attendant feature of section 457 plans is that they may provide less security to participants than do qualified plans.
How well did you know this?
1
Not at all
2
3
4
5
Perfectly
25
Q

money-purchase pension plan

A

In a money-purchase pension plan the investment risk is on the employees and thus an increase or decrease in the investments has no impact on contributions.
If the company gave everyone a raise then that would increase the company’s contributions.
If a key employee retired and two employees left that would decrease the company’s contributions.
Since the forfeiture allocations were allocated to the participants they would have no effect of the company’s contributions.

How well did you know this?
1
Not at all
2
3
4
5
Perfectly
26
Q

What’re the limits to SEP-IRA (SIMPLIFIED EMPLOYEE PENSION (SEP) PLAN)?

A
  1. The limits for contributions are the lesser of the following:
    a. 25% of compensation [covered compensation is limited to $285,000 (2020)]
    b. $57,000 (2020)
  2. The SARSEP plan salary reduction contribution limit for 2020 is $19,500 (the same as the limit for 401(k) plans).
    a. Individuals who have attained age 50 may make additional catch-up contributions.
    b. The additional catch-up amount is $6,500 (2020
How well did you know this?
1
Not at all
2
3
4
5
Perfectly
27
Q

Which of the following accurately describes the similarities between a traditional Individual Retirement Account and a SEP-IRA?

I. Individual ownership of the account.

II. All contributions into the account are fully owned by participant.

III. Subject to early withdrawal penalties and minimum distribution regulations.

IV. All distributions from plan taxed as ordinary income.

A)I and III only.
B)II and IV only.
C)II, III and IV only.
D)I, II, III and IV.

A

Rationale
The correct answer is “D.” All of these characteristics are shared by traditional IRAs and SEP-IRAs. Any non-deductible contribution to an IRA are taxed as a pro-rata distribution.

How well did you know this?
1
Not at all
2
3
4
5
Perfectly
28
Q

Shane’s Rib Shack has a Target Benefit Plan. They have 10 employees with the following compensations: Employee Compensation

1- $300,000

2- $100,000

3- $75,000

4- $50,000

5- $50,000

6- $50,000

7- $50,000

8- $25,000

9- $25,000

10- $20,000

Based on the actuarial table that was established at the inception of the plan they should fund the plan with $210,000. What is the maximum deductible contribution that can be made to the plan?

A
A)$182,500
Rationale
The correct answer is "A." Since the plan is a defined contribution plan the maximum deductible contribution is 25% of the total covered compensation. The max covered compensation of all employees is $730,000. Thus the maximum deductible limit is $182,500 ($730,000 x 25%). Remember to limit employee 1 to the $285,000 (2020) covered compensation limit. The actuarial table amount is irrelevant because this a defined contribution plan.
B)$186,250
C)$195,000
D)$210,000
How well did you know this?
1
Not at all
2
3
4
5
Perfectly
29
Q

Why life insurance in qualified plans is subject to income?

A

When life insurance is purchased in a qualified account, the premium is paid with pretax dollars. Consequently, the participant must recognize the economic benefit received as taxable income. … The remaining cash value can remain in the plan or be taxed as a qualified plan distribution.

How well did you know this?
1
Not at all
2
3
4
5
Perfectly
30
Q

Which of the following is correct about the life insurance in a qualified plan?

A
  1. The policy will be included in his gross estate if he were to die while still working.
  2. Part of the proceeds could be taxable to his beneficiary if it is a cash value policy.
  3. When he distributes the policy from his plan at retirement, he can convert it to an annuity within 60 days to avoid taxation.
How well did you know this?
1
Not at all
2
3
4
5
Perfectly
31
Q

George, age 35, works for XZY Brothers, Inc., which is installing a new SIMPLE IRA plan in the current year with the maximum match for this year. George makes $30,000 per year and is eligible to participate in the plan. Which of the following is true?

A

George can put in 100% of salary up to $13,500 (2020). XZY will match dollar for dollar up to 3% of salary ($30,000 x .03 = $900). So a total of $14,400 will be placed into the account.

SIMPLE IRA: Savings incentive match plan for employees.

An employer-sponsored individual retirement account or individual retirement annuity arrangement that is similar from a contribution perspective to a qualified profit-sharing plan

  1. A SEP plan must cover all employees who are at least age 21 and who have worked for the employer during three of the preceding five calendar years.
How well did you know this?
1
Not at all
2
3
4
5
Perfectly
32
Q

To retain its qualified status, a retirement plan must:
I. Have pre-death and post-death distributions.

II. Stipulate rules under what circumstances employee contributions are forfeited.

III. Be intended to be permanent.

IV. Be established by the employer.

A
A)I and II only.
B)II, III and IV only.
C)I, III and IV only.
Rationale
The correct answer is "C." Employee contributions must be vested and cannot be required to be forfeited.
D)I, II, III and IV.
How well did you know this?
1
Not at all
2
3
4
5
Perfectly
33
Q

Cafeteria plans have which of the following characteristics?

I. Must offer a choice between at least one qualified “pre-tax” benefit and one non-qualified “cash” benefit.

II. Medical Flexible Spending Accounts (FSAs) can reimburse medical expenses not covered by insurance for the participant and all dependents.

III. Changes in election amount during the plan year can only occur with a “qualifying change in family status.”

IV. Salary reductions are not subject to income taxes but payroll taxes apply.

A

A)I, II and IV only.
B)II, III and IV only.
C)I, II and III only.

Rationale
The correct answer is “C.” Cafeteria Plans (Section 125) allow salary reductions which are taken from an employee’s salary before Federal and State withholding tax as well as Social Security and Medicare taxes (FICA).

At least one taxable and non-taxable benefit must be offered under a plan.
Medical FSAs allow reimbursement for eligible medical expenses for the employee and any dependents.
A qualifying change in status is required to make a mid-year change in elections.

D)I, II, III and IV.

How well did you know this?
1
Not at all
2
3
4
5
Perfectly
34
Q

what’s the difference between payroll taxes and income taxes

A

Payroll tax is a percentage of an employee’s pay. Income tax is made up of federal, state, and local income taxes. … Income tax amounts are based on a number of factors, such as an employee’s Form W-4 and filing status.

The difference between payroll tax and income tax also comes down to what the taxes fund. Whereas income taxes go to a general government fund, payroll taxes specifically go to Social Security and Medicare funds.

How well did you know this?
1
Not at all
2
3
4
5
Perfectly
35
Q

Who are favored under a defined benefit plan?

A

Long-term employees are favored under a defined benefit plan

How well did you know this?
1
Not at all
2
3
4
5
Perfectly
36
Q

dues to business-related organizations provided as a fringe benefit are:

A

A)Includable in taxable income of all covered employees.
B)Includable in the taxable income of key employees only.
C)Excludable from the taxable income of all covered employees.
Rationale
The correct answer is “C.” Dues and licenses are excluded from taxable income if directly related to the employee’s job.
D)Excludable from the taxable income of non-highly compensated employees only.

How well did you know this?
1
Not at all
2
3
4
5
Perfectly
37
Q

Dues

A

n. 会费

How well did you know this?
1
Not at all
2
3
4
5
Perfectly
38
Q

what’s the group term life insurance coverage?

A

group term life insurance coverage is $50,000.

How well did you know this?
1
Not at all
2
3
4
5
Perfectly
39
Q

what is the maximum excess rate for integrated stock bonus plan?

A

The maximum excess rate is 2 times the contribution rate limited to a disparity of 5.7%

How well did you know this?
1
Not at all
2
3
4
5
Perfectly
40
Q

Which of the following is a correct statement about the income tax implications of employer premium payments for group health insurance?

A

A)An S Corporation can only deduct 70% of the premiums for all employees.
B)In a sole proprietorship, the premiums for both the owner and the non-owner are fully deductible.
C)If stockholder/employees of a closely held C corporation are covered as employees, the premiums are fully deductible.
Rationale
The correct answer is “C.” S Corporations and proprietorships cannot deduct any premiums for group health insurance for owners. Non-owner employee health premiums are fully deductible to both entities. Answer “D” is incorrect because partners are able to deduct 100% of the health insurance premium on their individual tax returns.
D)Premium costs paid by a partnership are passed through to the partner, who can deduct 70% of the costs on their individual tax returns

How well did you know this?
1
Not at all
2
3
4
5
Perfectly
41
Q

group term life

A

The contract has a master group policy.

How well did you know this?
1
Not at all
2
3
4
5
Perfectly
42
Q

group whole life program

A

The coverage is based on a combination of decreasing units of group term and accumulating units of single premium whole life

How well did you know this?
1
Not at all
2
3
4
5
Perfectly
43
Q

group universal life insurance?

A
  1. Expenses are often lower than for individual universal life policies.
  2. These policies offer the potential for higher returns than whole life policies.
How well did you know this?
1
Not at all
2
3
4
5
Perfectly
44
Q

Which employee fringe benefits would be taxable to the employees?

A

Monthly dues to a health club paid by the employer.

must be provided “on the employer premises.”

How well did you know this?
1
Not at all
2
3
4
5
Perfectly
45
Q

Minimum funding requirements apply to?

A

Minimum funding requirements apply to pension plans, not profit sharing plans.
Forfeitures in profit sharing plans are usually allocated to the remaining participants’ individual accounts.

How well did you know this?
1
Not at all
2
3
4
5
Perfectly
46
Q

“use-it-or-lose-it” provision

A

Cafeteria plans have a “use-it-or-lose-it” provision which requires any funds not used to pay qualified claims during the plan year be forfeited back to the plan sponsor. Forfeited funds cannot be rebated back to the individual employee who forfeited the funds.

1) A separate FSA salary reduction must be made for each type of eligible benefit.
2) A salary reduction for an FSA will lower an employee’s income for social security tax purposes if the employee earns less than the social security wage base.
3) An FSA is technically a cafeteria plan benefit that can be used by itself or as part of a broader cafeteria plan.

How well did you know this?
1
Not at all
2
3
4
5
Perfectly
47
Q

What is TSA plan?

A

A 403(b) plan (tax-sheltered annuity plan or TSA) is a retirement plan offered by public schools and certain charities. … Salary contributed to a Roth account is taxed currently, but is tax-free (including earnings) when distributed

How well did you know this?
1
Not at all
2
3
4
5
Perfectly
48
Q

Which funding vehicles are eligible (approved) for TSAs?

A

TSA can be invested in annuities and mutual funds.
I. Fixed Annuity Contracts.

II. Mutual funds.

III. Variable annuity contracts.

How well did you know this?
1
Not at all
2
3
4
5
Perfectly
49
Q

Describe a short-term disability plan

A
  1. Premiums under an employer-paid plan are deductible to the employer when paid to the insurance company.
  2. The employee must claim the benefits from the employer-paid policy as taxable income.
  3. If employees pay for the premium on an after-tax basis, benefits are tax exempt.
  4. Premiums paid by the employee are deductible only though a Section 125 cafeteria plan, then benefits are taxable.
  5. Definitions of disability are much more liberal under short-term disability than under long-term disability.
How well did you know this?
1
Not at all
2
3
4
5
Perfectly
50
Q

Group term life insurance

A
  1. A low-cost,
  2. tax-advantaged policy
  3. may be either contributory or non-contributory
  4. does not require a medical examination
How well did you know this?
1
Not at all
2
3
4
5
Perfectly
51
Q

Group paid-up life insurance

A

It combines both term life insurance and whole life insurance

How well did you know this?
1
Not at all
2
3
4
5
Perfectly
52
Q

Group ordinary life insurance

A

It contains the cash value

How well did you know this?
1
Not at all
2
3
4
5
Perfectly
53
Q

What are two methods that Defined benefit pension plans have by which they can be integrated with Social Security?

A

the excess method and the offset method.

a. Excess method—The plan defines a level of compensation, called the integration level, and then provides a higher rate of contribution and benefits for compensation above the integration level.
b. Offset method—A fixed amount or a formula amount that is designed to represent the existence of Social Security benefits reduces the plan formula
c. The Money Purchase Pension plan is a Defined Contribution Plan and must use the excess method. Simple’s and ESOPs cannot be integrated with Social Security.

How well did you know this?
1
Not at all
2
3
4
5
Perfectly
54
Q

Describe group universal life insurance plan

A

A)It allows employees to borrow or withdraw cash.
B)It provides an opportunity to continue coverage after retirement.
C)The entire premium cost is paid by the employee.
the employee is required to pay part or all of the premium cost of group universal life insurance.
D)It provides flexibility in designing coverage to best meet individual needs.

How well did you know this?
1
Not at all
2
3
4
5
Perfectly
55
Q

Lodging is only excluded in taxable income of all covered employees.

A

Lodging is only excluded if :
(1) it is furnished on the business premises (the place of work),
(2) it is furnished for the employER’s convenience, and (3) the employee accepts it as a condition of employment.
A vacation stay does not meet any of the these criteria so it is fully taxable for all covered employees.

How well did you know this?
1
Not at all
2
3
4
5
Perfectly
56
Q
Match the following statement with the type of retirement plan that it most completely describes: " A defined benefit plan that has the appearance of a defined contribution plan" is a...
A)Profit sharing plan.
B)Money purchase plan.
C)SIMPLE IRA.
D)Cash balance plan.
A

Rationale
The correct answer is “D” - Cash balance plan. Answers “A” and “C” are incorrect since they are not defined benefit plans. Answer “B” - Money purchase plan is a “pension plan” but it does not provide employees with a defined benefit, only a defined contribution. Answer “D” - Cash balance plan provides a defined benefit (returns are guaranteed by the employer) and the employee receives an “account” to see how much they have.

How well did you know this?
1
Not at all
2
3
4
5
Perfectly
57
Q

SIMPLE IRA

A

requires an employer match and is not a qualified plan

How well did you know this?
1
Not at all
2
3
4
5
Perfectly
58
Q

A Money purchase plan and A Defined benefit plan

A

are all pension plans

How well did you know this?
1
Not at all
2
3
4
5
Perfectly
59
Q

a pension plan has the following characteristics:

A

A)Benefits must be definitely determinable under a formula for employer contribution or a formula for retirement benefits.
B)Benefits may not be withdrawn prior to termination of employment or retirement.
C)Contributions and benefits are determined in the long run by the profits of the employer.
Rationale
The correct answer is “C.” Employer contributions may be made without regard to company profits or retained earnings. Answers “A”, “B” and “D” are requirements for all qualified plans.
D)Generally pays retirement benefits over a period of years.

How well did you know this?
1
Not at all
2
3
4
5
Perfectly
60
Q

tax withholding

A

预扣税

How well did you know this?
1
Not at all
2
3
4
5
Perfectly
61
Q

Like-kind properties

A

Like-kind properties must be traded in the US for the industrial use and are real estate assets of a similar nature that can be exchanged without incurring any tax liability under Section 1031 of the Internal Tax Code. Properties must be held for business or investment purposes but do not need to be similar in grade or quality.

How well did you know this?
1
Not at all
2
3
4
5
Perfectly
62
Q

Individual Retirement Accounts (IRA)?

A

I. Distributions to the IRA owner must begin by April 1 of the year following the year in which the owner reaches age 70 1/2 (if by 12/31/2019) or age 72 (if 70 1/2 after 12/31/2019).

II. After the owner’s death, the entire amount remaining in the IRA is included in the owner’s gross estate for federal estate tax purposes.

funds distributed from an IRA are always treated as ordinary income, regardless of source and 5 year forward averaging is no longer available for any distribution.
all distributions from an IRA not meeting the statutory exemptions are subject to the premature distribution penalty, regardless of source.

How well did you know this?
1
Not at all
2
3
4
5
Perfectly
63
Q

Matt is a participant in a profit sharing plan which is integrated with Social Security. The base benefit percentage is 6%. Which of the following statements is/are true?

I. The maximum permitted disparity is 100% of the base benefit level or 5.7%, whichever is lower.

II. The excess benefit percentage can range between 0% and 11.7%.

III. Elective deferrals may be increased in excess of the base income amount.

IV. The plan is considered discriminatory because it gives greater contributions to the HCEs.

A

A)I and II only.
Rationale
The correct answer is “A.” His base rate is 6% and the social security maximum disparity is 5.7% for 11.7% as the top of his range.

Statement “III” is incorrect because integration does not affect voluntary deferrals by employees. Statement “IV” is incorrect because, done properly, integration is NOT considered discriminatory.
B)I, II and IV only.
C)II only.
D)I, II, III and IV.

How well did you know this?
1
Not at all
2
3
4
5
Perfectly
64
Q

457 Plan

A

457 plans are IRS-sanctioned, tax-advantaged employee retirement plans. They are offered by state, local government and some nonprofit employers. Participants are allowed to contribute up to 100% of their salary, provided it does not exceed the applicable dollar limit for the year.

How well did you know this?
1
Not at all
2
3
4
5
Perfectly
65
Q

Which of the following statements accurately describes the requirements for a plan established under Section 457 to be qualified?

I. Distributions are NOT permitted until age 70 1/2 or termination of employment if before 59 1/2.

II. To avoid constructive receipt, the agreement must be signed during the same month the services are rendered and prior to receipt of the paycheck.

III. Eligible participants include employees of agencies, instrumentalities, and subdivisions of a state, as well as Section 501 tax-exempt organizations.

IV. The maximum employee elective deferral excluding catch-ups is limited to $19,500 (2020) (as indexed), or 100% of includible compensation.

A
A)I and II only.
B)I and III only.
C)II and IV only.
D)III and IV only.
Rationale
The correct answer is "D." Statement "II" is incorrect because the agreement must be signed PRIOR to the month the services are rendered. Statement "I" is incorrect because distributions are permitted at termination or normal retirement age as stated in plan document (not 70 1/2). The maximum elective deferral including the catch-up is $39,000 for 2020, excluding the catch-up is $19,500 for 2020.
How well did you know this?
1
Not at all
2
3
4
5
Perfectly
66
Q

money purchase plan

A

defined contribution limit of $57,000. (25% of $230,000 is $57,500 but the single employer limit applies and is $57,000 for 2020).

How well did you know this?
1
Not at all
2
3
4
5
Perfectly
67
Q
A hybrid plan that uses a discretionary contribution but adjusts for age is a form of a:
A)Profit sharing plan.
B)Money purchase plan.
C)Cash balance plan.
D)Defined benefit plan.
A
A)Profit sharing plan.
Rationale
The correct answer is "A." Answers "B," "C" and "D" all require minimum contribution levels. Answer "A" - Profit sharing plan only requires that contributions be "substantial and recurring." More specifically, an age-based profit sharing plan would be correct.
B)Money purchase plan.
C)Cash balance plan.
D)Defined benefit plan.
How well did you know this?
1
Not at all
2
3
4
5
Perfectly
68
Q

Your client, a 35% owner of a regular C corporation, wants to take out a loan from the company sponsored profit-sharing plan. In order for the loan not be a prohibited transaction, which of the following conditions must apply:

I. Loans are available to all participant/beneficiaries on a reasonably equivalent basis.

II. Have a reasonable rate of interest.

III. Made in accordance with specific plan provisions.

IV. Must be adequately secured.

A)II only.
B)I, II and IV only.
C)None of the above.
D)All of the above.

A

Rationale
The correct answer is “D.” This is an owner of a regular corporation (C Corporation.) Owners of C Corporations are eligible for loans as long as the safe-harbor rules are maintained. The items listed in Statements “I,” “II,” “III,” and “IV” are the safe-harbor rules.

How well did you know this?
1
Not at all
2
3
4
5
Perfectly
69
Q
Sherman, age 52, works as an employee for Cupcakes Etc, a local bakery. Cupcakes sponsors a 401(k) plan. Sherman earns $50,000 and makes a 10% deferral into his 401(k) plan. His employer matches the first 3% deferral at 100% and they also made a 5% profit sharing contribution to his plan. Sherman also owns his own landscaping business and has adopted a solo 401(k) plan. His landscaping business earned $40,000 for the current year. What is the most that Sherman can contribute in the solo plan, assuming his self-employment taxes are $6,000?
A)$19,500
B)$21,000
C)$26,000
D)$28,400
A

Rationale
The correct answer is “D.” An individual can defer up to $19,500 (2020) plus an additional $6,500 catch up for all of their 401(k) and 403(b) plans combined. Since he is 50 or older he can contribute the 19,500 + 6,500 = $26,000. Since he already contributed $5,000 into his employer plan he can still defer $21,000 ($26,000 - $5,000) into the solo plan. The employer contributions in this question are in addition to the employee deferral limit.

Employer contribution into the solo plan:

i) self-employment income $40,000
ii) less 1/2 SE tax $3,000
iii) Net $37,000
iv) X 20%
v) employer contribution $7,400

Total contribution to the solo plan = $21,000 + $7,400

How well did you know this?
1
Not at all
2
3
4
5
Perfectly
70
Q

This plan can provide for voluntary participant contributions which must be matched by the employer.”

A

SIMPLE IRA

How well did you know this?
1
Not at all
2
3
4
5
Perfectly
71
Q

Which plans don’t permit employee elective deferrals?

A

Money purchase plan.

Defined benefit plan.

How well did you know this?
1
Not at all
2
3
4
5
Perfectly
72
Q

Which plan doesn’t require an employer match?

A

Profit sharing plan with a 401(k) component.

How well did you know this?
1
Not at all
2
3
4
5
Perfectly
73
Q

Section 121

A

IRC section 121 allows a taxpayer to exclude up to $250,000 ($500,000 for certain taxpayers who file a joint return) of the gain from the sale (or exchange) of property owned and used as a principal residence for at least two of the five years before the sale. A taxpayer can claim the full exclusion only once every two years. A reduced exclusion is available to anyone who does not meet these requirements because of a change in place of employment, health or certain unforeseen circumstances.

How well did you know this?
1
Not at all
2
3
4
5
Perfectly
74
Q

standard deduction?

A

standard deduction is $24,800.
They are not age 65 or older so they don’t receive an additional standard deduction. They do not receive an additional standard deduction for Marie’s blindness because additional standard deductions for age and blindness are allowed only for the taxpayer and spouse, and not for their dependents. ————–> taxable income is equal to their gross income less deductions for adjusted gross income less the greater of the standard deduction or itemized deductions, and less personal and dependency exemptions. —————————-> Ursula must use the Single filing status. In addition, she is entitled to one additional standard deduction because of her blindness. Therefore, her standard deduction for the current year is $14,050 ($12,400 + $1,650).

How well did you know this?
1
Not at all
2
3
4
5
Perfectly
75
Q

Cafeteria plan:

A

A cafeteria plan must offer at least one taxable benefit, usually cash, and one qualified nontaxable benefit. —–>.Childcare provided under a cafeteria plan is not eligible for the dependent care credit.——>Cafeteria plans (Section 125) a. The employer offers the employee the choice between cash or selected nontaxable benefits. b. If the employee chooses the benefit, it remains nontaxable. c. If the employee chooses cash equal to the cost of the benefit, the cash is included in income. d. This can also give employees the opportunity to buy certain benefits with after-tax contributions.

How well did you know this?
1
Not at all
2
3
4
5
Perfectly
76
Q

passive activity in rental:

A

a. Unless excepted by the Code, all rental activities are deemed passive. 1.) Rental activities occur when payments are received for the use of tangible property. 2.) If an activity is an exception in the Tax Code, it still must pass the material par-ticipation test to be considered active. b. An activity is not considered a rental under any of the following six conditions: 1.) The average customer use is seven days or less. 2.) The average customer use is more than seven days but not over 30 days, and the owner provides significant personal services. 3.) The owner also provides extraordinary services (rental period is irrelevant). 4.) The rental activity is incidental to a nonrental activity of taxpayer (investment or trade/business). 5.) The property is customarily made available during business hours for nonexclu-sive use by customers (e.g., golf course). 6.) The property is used in a nonrental activity conducted by a partnership, S cor-poration, or joint venture, where the owner has an interest.

How well did you know this?
1
Not at all
2
3
4
5
Perfectly
77
Q

Which of the following is true concerning IRA contributions?
A)An employee who makes voluntary contributions to a 401(k) plan is not considered an active participant.
B)An employee who receives no contributions or forfeiture allocations in their employer’s profit sharing plan is not considered an active participant.
C)An employee who makes no voluntary contributions to a thrift plan yet receives forfeiture allocations to a profit sharing plan is not considered an active participant.
D)An employee participating in a Section 457 plan is considered an active participant if employee pretax deferrals are elected.

A

B)An employee who receives no contributions or forfeiture allocations in their employer’s profit sharing plan is not considered an active participant.
Rationale
The correct answer is “B.” Answers “A” and “C” are conditions of being considered an active participant. Answer “D” is incorrect because 457 plan participants are not considered active participants for IRA contribution purposes.

How well did you know this?
1
Not at all
2
3
4
5
Perfectly
78
Q

Which of the following tasks are the primary responsibilities of a plan trustee?

I. Determining which employees are eligible for participation in the plan, vesting schedule, and plan benefits.

II. Preparing, distributing, and filing reports and records as required by ERISA.

III. Investing the plan assets in a “prudent” manner.

IV. Monitoring and reviewing the performance of plan assets.

A)I and III only.
B)I and II only.
C)II and IV only.
D)III and IV only.

A

Rationale
The correct answer is “D.” The duties explained in Statements “I” and “II” are responsibilities of the plan administrator.

How well did you know this?
1
Not at all
2
3
4
5
Perfectly
79
Q

Does the profit sharing plan require the certain percentage of the match?

A

The match for a profit sharing plan with 401(k) provisions can vary every year and there is no required percentage. However, a SIMPLE can vary the match in only 2 of 5 years.

How well did you know this?
1
Not at all
2
3
4
5
Perfectly
80
Q

In order for a group term life insurance plan to be non-discriminatory, which of the following is true?

A)At least 80% of all employees must benefit from the plan.
B)At least 85% of the participants must be non-highly compensated employees.
C)If the plan is part of a cafeteria plan, the plan must comply with the non-discrimination rules of Section 125.
D)The bottom band of benefits must be no less than 10% of the top band with no more than a 2 times differential between bands.

A

Rationale
The correct answer is “C.” A plan must benefit 70% of all employees or a group of which at least 85% are not key employees.
If the plan is part of a cafeteria plan, it must comply with Section 125 rules. The difference between the bands in “D” must be no greater than 2.5 times the next smaller band with the bottom band being equal to no less than 10% of the top band.

How well did you know this?
1
Not at all
2
3
4
5
Perfectly
81
Q

What does the benefit of employer-provided short-term disability plan offer?

A
  1. Short-term disability benefits usually start the eighth day of an illness (first day for an accident); and
  2. Generally last no more than six months. (not one year)
How well did you know this?
1
Not at all
2
3
4
5
Perfectly
82
Q

Which of the following statements accurately reflects the overall limits and deductions for employer contributions to qualified plans?

I. An employer’s deduction for contributions to a money purchase pension plan and profit sharing plan is limited to the lesser of 25% of covered payroll or the maximum Section 415 limits permitted for individual account plans.

II. An employer’s deduction for contributions to a defined benefit pension plan and profit sharing plan cannot exceed the lesser of the amount necessary to satisfy the minimum funding standards or 25% of covered payroll.

III. Profit sharing minimum funding standard is the lesser of 25% or the Section 415 limits permitted for individual account plans.

A)I only.
B)I and II only.
C)II and III only.
D)I, II and III.

A

Rationale
The correct answer is “A.” Statement “II” is incorrect because there is no 25% of covered payroll limitation in a DB plan. Statement “III” is incorrect because there is no minimum funding standard for profit sharing plans.

How well did you know this?
1
Not at all
2
3
4
5
Perfectly
83
Q

Defined benefit pension plans increase the funding costs associated with the plan if :

A

If turnover is reduced, then funding will increase. If the retirement age is reduced, funding will increase as more years will need to be funded. If the rate of return increases, then funding will decrease. If wages increase, then funding will increase.

How well did you know this?
1
Not at all
2
3
4
5
Perfectly
84
Q

What are the AGI limitation for Traditional IRA and a Roth IRA (2020)?

A

Traditional IRA since she is below the AGI limitation for a single active participant ($65,000 - $75,000) (2020). She can also contribute to a Roth IRA because she is below the AGI limitation of $124,000 - $139,000 (2020)

How well did you know this?
1
Not at all
2
3
4
5
Perfectly
85
Q

Is there an age limit for Traditional (SECURE Act 2019) or Roth IRAs.

A

there is no age limit for Traditional (SECURE Act 2019) or Roth IRAs.

How well did you know this?
1
Not at all
2
3
4
5
Perfectly
86
Q
Dr. Woods, age 29, is a new professor at Public University (PU) where he has a salary of $111,000. PU sponsors a 403(b) plan and a 457 plan. Dr. Woods also has a consulting practice called Damage Estimate Claims (DEC). He generates $200,000 of revenue and has $50,000 of expenses for DEC. Assume his self-employment tax is $19,790. What is the most that he could contribute to all of the retirement plans this year assuming he establishes a Keogh plan for DEC?
A)$39,000
B)$57,000
C)$67,021
D)$73,521
A

Rationale
The correct answer is c.
Dr. Woods can contribute $19,500 to each of the 403(b) plan and the 457 plan. In addition, he can establish a Keogh plan and contribute 20% of his net self-employment income after deducting ½ self,-employment taxes.
The 403(b) and 457 can both receive 19,500 (qualified and deferred comp plans).

The Keogh needs to follow self-employment contribution rules - see the retirement pre-study book)

$200,000 of income for DEC

-$50,000 of expenses for DEC

150,000 Net income

-9,895 (Assume his self-employment tax is $19,790, use ½)

140,105

X 20% (contribution rate / (1+contribution rate) = self-employed contribution rate)

28,021

In total 19,500 + 19,500 + 28,021 = 67,021

How well did you know this?
1
Not at all
2
3
4
5
Perfectly
87
Q

What is Keogh plan?

A

A Keogh plan is a tax-deferred retirement plan designed for self-employed people. Keogh plans get their name from the man who created them, Eugene Keogh. He established the Self-Employed Individuals Tax Retirement Act of 1962, also known as the Keogh Act.

How well did you know this?
1
Not at all
2
3
4
5
Perfectly
88
Q

Is the statement right “The amount needed for the investment pool to fund period certain annuities for each participant upon retirement.” ?

A

No. DB plans deal with life annuities, NOT period certain annuities.

How well did you know this?
1
Not at all
2
3
4
5
Perfectly
89
Q

What conditions must be met by Loans from a qualified plan?

A

Limit on loans is the lesser of 50% of the account value or $50,000 for a period of five years, unless for a primary residence.

How well did you know this?
1
Not at all
2
3
4
5
Perfectly
90
Q

Which plan should be suitable for wide fluctuations in cash flow?

A

Wide fluctuation in cash flow call for a profit sharing plan.

How well did you know this?
1
Not at all
2
3
4
5
Perfectly
91
Q

Are the Target benefit plans and defined benefit plans pension plans?

A

Both plans are pension plans
I. Minimum funding standards apply.

II. Qualified joint and survivor annuity requirements apply.

How well did you know this?
1
Not at all
2
3
4
5
Perfectly
92
Q

Which plan might be good for “ High investment earnings increase participant retirement benefits?

A

It applies only to the target benefit plan (because it is a DC plan) NOT the DB plan.

How well did you know this?
1
Not at all
2
3
4
5
Perfectly
93
Q

PBGC

A

Pension Benefit Guaranty Corporation

How well did you know this?
1
Not at all
2
3
4
5
Perfectly
94
Q

How to describe the eligible individual accounts?

A

Eligible individual accounts (usually associated with profit sharing, 401(k) and ESOP) are defined contribution plans, and are not subject to PBGC, nor minimum funding requirements.

How well did you know this?
1
Not at all
2
3
4
5
Perfectly
95
Q

How to describe A key employee?

A

A key employee is an individual who (1) owns more than 5% of the business, (2) is an officer with compensation greater than $185,000 (2020), or (3) owns greater than 1% of the business and has compensation greater than $150,000.

How well did you know this?
1
Not at all
2
3
4
5
Perfectly
96
Q

What does “2-6 year graded vesting” mean?

A

After two years, the employee would be 20% vested, after three years, 40%, with the employee eventually becoming fully vested after six years.

How well did you know this?
1
Not at all
2
3
4
5
Perfectly
97
Q

Which year does the only vesting schedules allow?

A

2-year cliff vesting.
3-year cliff vesting.
and 2-6 year graded vesting

How well did you know this?
1
Not at all
2
3
4
5
Perfectly
98
Q

Does a housewife can contribute to her IRA?

A

Even though she does not have any earned income of her own, she can use Brad’s earnings to qualify for the contribution.

How well did you know this?
1
Not at all
2
3
4
5
Perfectly
99
Q

The Health Insurance Portability and Accountability Act of 1996 (HIPAA) impacts an employee and employer in which of the following ways:

I. An employee without creditable coverage can generally only be excluded by the group health insurance plan (if offered) for up to twelve months.

II. The waiting period is reduced by the amount of “creditable coverage” at a previous employer.

III. If the employee does not enroll in the group health insurance plan at the first opportunity, an 18-month exclusion period may apply.

A)I and II only.
B)I, II and III only.
C)II and III only.
D)II only.

A

Rationale
The correct answer is “B.” All three statements are true. If you have a pre-existing condition that can be excluded from your plan coverage, then there is a limit to the pre-existing condition exclusion period that can be applied. HIPAA limits the pre-existing condition exclusion period for most people to 12 months (18 months if you enroll late), although some plans may have a shorter time period or none at all. In addition, some people with a history of prior health coverage will be able to reduce the exclusion period even further using “creditable coverage.” People with a history of prior health coverage will be able to reduce the exclusion period even further using “creditable coverage.”

How well did you know this?
1
Not at all
2
3
4
5
Perfectly
100
Q

IRC Section 415(c) applies an “annual addition” limited to certain qualified plans. Which of the following statements is correct?

I. The limit is the lessor of 25% of compensation or $57,000 for the current year.

II. The limit only applies to defined contribution plans.

III. Includable additions include forfeiture reallocations, employer and employee contributions and investment earnings.

IV. Salary deferrals are included as part of the annual additions limit.

A)II only.
B)I and II only.
C)II and III only.
D)II and IV only.

A

Rationale
The correct answer is “D.” Statement “I” is incorrect as the limit is the lesser of 100% of compensation or the annual limit. Statement “III” is incorrect because investment earnings are never included in the Section 415(c) limit calculation. Statement “IV” is correct as salary deferral contributions by employees is counted against the IRC 415(c) limit.
E)II, III and IV only.

How well did you know this?
1
Not at all
2
3
4
5
Perfectly
101
Q

ERISA

A

The Employee Retirement Income Security Act of 1974 (ERISA) is a federal law that sets minimum standards for most voluntarily established retirement and health plans in private industry to provide protection for individuals in these plans.

How well did you know this?
1
Not at all
2
3
4
5
Perfectly
102
Q

Which transactions between a disqualified person and a qualified plan would be considered prohibited transactions under ERISA?

I. The employer purchases a mortgage note which is currently in default for more than the fair market value.

II. The employer sells a piece of raw (undeveloped) land to the qualified plan for a price substantially below fair market value.

III. Loan to a 100% owner/participant on the same basis as every other participant as set forth in the plan documents.

IV. The purchase of employer stock for full and adequate consideration by a 401(k) plan.

A)I and II only.
C)I, II and III only.
D)I, II and IV only.

A

Rationale
The correct answer is “A.” Any transaction between a disqualified person and the trust is considered a prohibited transaction. In Statement “I,” the employer could purchase the mortgage note at a markup to future market value, thus giving the pension (and consequently his own individual retirement account) a big boost in value, then sell the note to someone else and take a loss on their personal income tax. Thus, in essence making additional contributions to the plan. Statement “II” would accomplish the same purpose. Employer’s individual taxes would be reduced (lower profit on sale to the plan) but would have a dramatic increase in retirement plan assets.
B)II and III only.

How well did you know this?
1
Not at all
2
3
4
5
Perfectly
103
Q

The maximum service requirement that a profit sharing plan may impose as a condition of participation is:

A)1 year with semi-annual entry dates (1.5 years).
B)1 year with quarterly entry dates (1.25 years).
C)6 months.
D)2 years.

A

Rationale

The correct answer is “D.” This requires “immediate vesting.”

How well did you know this?
1
Not at all
2
3
4
5
Perfectly
104
Q

Which statement(s) accurately reflect(s) the Tax-Sheltered Annuity (TSA) provisions:

I. Salary reductions into a TSA are exempt from all payroll taxes.

II. The annual elective deferral limit may be increased by up to $3,000 for employees of certain organizations who have completed 15 years of service and meet certain other requirements.

III. Tax sheltered annuities must allow participants to invest in mutual fund, annuities and/or fixed income securities.

IV. To calculate the maximum exclusion allowance for make-up calculation purposes, the participant’s years of service and the amount of total excludable contributions made in the prior three years are needed.

A)I and II only.
B)II only.
C)I, III and IV only.
D)IV only.

A

Rationale
The correct answer is “B.” Statement “I” is incorrect because deferrals are still subject to Social Security and Medicare taxes. Statement “III” is incorrect because TSAs can only invest in mutual funds or annuities and not any direct investments. Statement “IV” is incorrect because the total excludable contributions must be for all prior years, not just the past three.

How well did you know this?
1
Not at all
2
3
4
5
Perfectly
105
Q

Which of the following accurately describes some attributes of non-qualified retirement plans?

I. The employee will pay Table 1 costs each year on an “employer pay all” split dollar life insurance arrangement.

II. The employer can deduct the premiums paid for a split-dollar life insurance arrangement in the year the premiums are paid.

III. Death benefits from a split-dollar arrangement, both the employer and the employee’s beneficiary’s share, are generally tax free.

IV. If the employee’s portion of the life insurance premium is greater than the P.S. 58 cost, the excess premiums “rolls forward” to a future year to accurately reflect the employee’s cost basis.

A)I and III only.
B)II and IV only.
C)I and IV only.
D)I and II only.

A

Rationale
The correct answer is “A.” Statement “II” is incorrect because the employer is unable to deduct any contributions to a non-qualified plan until the employee actually takes constructive receipt. In the traditional split-dollar arrangement, the employer has an interest in the cash values of the split-dollar policy equal to the amount of premiums paid, and therefore, there is never a deduction for premiums paid. Statement “III” - Because no tax deductions are taken for any premiums paid on the policy, the death benefits are tax-free. Statement “IV” - The employee is required to pay the Table 1 cost each year, without regard to premiums paid in previous years.

How well did you know this?
1
Not at all
2
3
4
5
Perfectly
106
Q
Larry and Terry plan to contribute a total of $2,900 to their IRAs for the current year. Larry has contributed $2,000 to his IRA and Terry will contribute $900. They both work outside the home and file a joint income tax return. Larry is a teacher at the local high school. His employer makes contributions into a 403(b) plan for Larry. Terry's employer makes contributions into her stock bonus plan account. Their modified AGI for the current tax year is $105,000. What is the combined maximum amount, if any, they are allowed to deduct for their IRA contribution?
A)$-0-
B)$2,500
C)$2,000
D)$2,900
A

Rationale
The correct answer is “D.” Reduction is (($105,000 - $104,000) / $20,000) x $6,000 = $300 (2020). Therefore, the maximum deduction is $6,000 - $300 = $5,700 each. Larry’s contribution of $2,000 and Terry’s contribution of $900 are both fully deductible.

Formula: Contribution x [(your AGI – the bottom of the phase out range) divided by the amount of the range (124,000-104,000 is where the 20k comes from)]

How well did you know this?
1
Not at all
2
3
4
5
Perfectly
107
Q

Which of the following is/are accurate of a Section 125 cafeteria plan?

I. 30% of the total benefits can accrue to key employees.

II. There must be at least one cash benefit.

III. Deferral of income is not allowed except through a 403(b).

IV. Salary reductions can be changed at any time during the year.

A)I only.
B)II only.
C)II and III only.
D)I and IV only.

A

Rationale
The correct answer is “B.” Statement “I” is incorrect because only 25% of the total benefits can accrue to key employees. Statement “III” is incorrect because deferrals are allowed only through a 401(k) plan. Statement “IV” is incorrect because mid-year changes in reductions are allowed only for qualified changes in status.

How well did you know this?
1
Not at all
2
3
4
5
Perfectly
108
Q

The Consolidated Omnibus Budget Reconciliation Act of 1985 (COBRA) mandates employers provide continuation coverage for former employees except under which of the following circumstances:

I. Employer has fewer than 20 employees.

II. Employee retires at the age of 65.

III. Death of the employee.

IV. Involuntary termination of employment due to gross misconduct.

A)I only.
B)III only.
C)I, III and IV only.
D)I, II and IV only.

A

Rationale
The correct answer is “D.” Employers must continue medical coverage to pay for final medical expenses after the death of the employee. Statements I, II, and IV are statutory exemptions to the COBRA requirement. (Note: A 65-year-old retiree would be covered under Medicare.)

Note: COBRA applies to all group health plans maintained by private-sector employers with 20 or more employees.

How well did you know this?
1
Not at all
2
3
4
5
Perfectly
109
Q

Describe 401(k) plans regarding Highly Compensated (HC) employees and Non-Highly Compensated (NHC) employees

A

A)Non-discriminatory rules state at least 80% of all NHCs must be covered by the plan, or the ratio of NHCs covered by the plan must be at least 80% of the HCs, or the average benefit percentage for NHCs is at least 80% of the benefit percentage of the HCs.

B)The Actual Deferral Percentage (ADP) Test limits employee voluntary contributions to the plan for the HCs. To accurately calculate the ADP, the administrator needs to know the percentage of income the NHCs contribute to the plan and how much the HCs defer to the plan.
Rationale
The correct answer is “B.” Option “A” would be correct if the percentages referenced were “70%” rather than “80%.” Option “C” is incorrect because only employees who are eligible to participate are included in the ADP test. Option “D” - At a NHC ADP level of 6%, the HC may not exceed 8% (6% + 2%). The 125% category does not start until NHC ADP level equals 8% or higher.

C)The Actual Deferral (ADP) Test takes all employees into account when calculating the test.

D)The average ratio of HCs may not exceed 125% of the ADP of the NHC group if the ADP of the NHC group is 6% or more.

110
Q

Martha has AGI of $1,000,000 (which is all comprised of earned income). She is single and age 45. She participates in her employer’s SIMPLE plan. Which of the following statements is true?

A)She can contribute $6,000 to a Traditional IRA and deduct all $6,000.
B)She can contribute $7,000 to a Traditional IRA and deduct all $7,000.
C)She can contribute $6,000 to a Traditional IRA and deduct $0.
D)She can contribute $7,000 to a Traditional IRA and deduct $0.

A

Rationale
The correct answer is “C.” Participating in a SIMPLE plan is considered being an “Active Participant.” She can contribute $6,000 (2020) to a Traditional IRA but cannot deduct any since she is above the AGI limitation for a single active participant ($65,000 - $75,000) (2020). She is unable to contribute to a Roth IRA because she is above the AGI limitation of $124,000 - $139,000 (2020). Because she is not 50 or older she is not allowed to make the $1,000 (2020) catch up contribution.

111
Q

Shawnte has AGI of $1,000,000 (which is all comprised of earned income). She is single and age 55. She participates in her employer’s 457 plan. Which of the following statements is true?

A)She can contribute $6,000 to a Traditional IRA and deduct all $6,000.
B)She can contribute $7,000 to a Traditional IRA and deduct all $7,000.
C)She can contribute $6,000 to a Traditional IRA and deduct $0.
D)She can contribute $7,000 to a Traditional IRA and deduct $0.

A

Rationale
The correct answer is “B.” Participating in a 457 plan is not considered being an “Active Participant.” She can contribute and deduct her contribution to a Traditional IRA $6,000 (2020) since she is not an active participant and therefore not subject to an AGI limitation. She is unable to contribute to a Roth IRA because she is above the AGI limitation of $124,000 - $139,000 (2020). Because she is 50 or older she is allowed to make the $1,000 (2020) catch up contribution.

112
Q

Which plans are non-qualified plans?

A

I. A 457 plan is a non-qualified deferred compensation plan of state and local government units and agencies and non-church-controlled tax-exempt organizations.

  1. Church-related organizations, such as schools, may be eligible.
  2. Contributions are pretax and funds grow tax deferred.
  3. There is no ERISA creditor protection for private 457 plans.

II. A 403(b) plan is a tax-advantaged plan, not a qualified plan.

113
Q

If a stock option is vested when it is received, and has a readily ascertainable value it is:
A)Assigned that value for taxation purposes.
B)Taxable when the stock is sold.
C)Taxable as soon as it is exercised.
D)Immediately taxable.

A

Rationale
The correct answer is “D.” Vested options are taxable based on the value of the option to the extent the Fair Market Value exceeds the option price.

114
Q

ascertainable

A

adj. 可确定的;可查明的

115
Q

Chris Barry, 59-years old, has been offered early retirement with an option of a two-year consulting contract. He has been a participant for the past 20 years in both the company defined benefit plan and defined contribution plan. His account balance is $120,000 in the profit-sharing plan and the present value of accrued benefit of the defined benefit plan is $240,000. Both provide for a lump sum distribution. Which of the following option(s) is/are available under the lump sum distribution rules?

I. Elect ten-year averaging on both plans.

II. Roll over the taxable portions of both plans to an IRA.

III. Elect long-term capital gains treatment on the DB plan.

IV. Elect five-year averaging on both plans.

A)I, II and III only.
B)I and II only.
C)II only.
D)IV only.

A

Rationale
The correct answer is “C.” Statement “I” is incorrect because he is not old enough for ten-year averaging. Statement “II” is correct because the taxable portion of any lump sum distribution may be rolled over into an IRA. Statement “III” is incorrect because he is not old enough to qualify for pre-74 capital gain treatment nor does he even have any actual pre-74 capital gain in the plan as he has only been in the plan for the last 20 years. Statement “IV” is incorrect because five-year averaging was repealed in 1999.

116
Q

pre-74 capital gain treatment

A

Capital gain treatment applies only to the taxable part of a lump-sum distribution resulting from participation in a qualified retirement plan before 1974. The amount treated as capital gain, reported on Form 1099-R, is taxed at 20%

117
Q

traditional IRA vs Roth IRA

A

For 2020, taxpayers can contribute a maximum of $6,000 for themselves and $6,000 for their spouses (working or nonworking), which is increased to $7,000 for those age 50 or older.
If the contribution is made to a traditional IRA, the contribution may be deductible. See Book 5—Retirement Savings and Income Planning, for other rules regarding the deductibility and limitations of contributions to a traditional IRA.
a. For taxpayers who are active participants in a qualified retirement plan, the ability to deduct a contribution to a traditional IRA phases out in 2020 at $65,000–$75,000 for single filers and $104,000–$124,000 for a taxpayer who is MFJ in 2020.
b. When only one spouse is an active participant, the nonparticipant spouse will have the deduction phased out at AGI levels (before considering the deductible IRA contribution) between and $196,000–$206,000 (in 2020).
c. The ability to make a contribution to a Roth IRA phases out at AGI levels as follows: Filing Status Married, joint (2020 $196,000–$206,000) Single ($124,000–$139,000) Married, separate $0–$10,000 2. Roth IRA contributions are never deductible.

118
Q

A parent-subsidiary group exists if the parent company owns what percentage of voting stock in another corporation?

A

At least 80%. This is important because parent-subsidiary companies must have substantially equal benefits or cover employees of all subsidiary companies under the same plan.

119
Q

Simplified Employee Pension plan (SEP)

A

A simplified employee pension (SEP) is an individual retirement account (IRA) that an employer or self-employed individual can establish. SEP IRAs are used by small businesses and self-employed individuals to meet their retirement savings needs

120
Q

Which one of the following is a possible disadvantage of a Simplified Employee Pension plan (SEP) for an employer?

A)The SEP’s trustee is subject to ERISA’s prohibited transaction excise tax penalties.
B)A SEP must have a fixed contribution formula that is non-discriminatory.
C)SEPs prohibit forfeitures.
D)Employer contributions to a SEP are subject to payroll taxes.

A

Rationale
The correct answer is “C.” Options “A” and “D” are false. Option “B” is a mandatory characteristic of all qualified defined contribution pension plans but not profit sharing plans.

121
Q

Can Qualified Domestic Relations Order (QDRO) be rolled over into an IRA?

A

a QDRO may be rolled over into an IRA as long as the rollover is accomplished within 60 days of the distribution.

122
Q

Does a 401(k) plan is allow voluntary employee elective deferral contributions?

A
401(k) plan is the only option which allows voluntary employee elective deferral contributions, as desired by the two assistants.
vs. 
Money purchase pension plan.
Defined benefit plan.
Profit-sharing plan.
123
Q

immaterial

A

adj. 非物质的;无形的;精神的;不重要的

124
Q

Is the statement right ? “In a discriminatory plan, a highly-compensated employee must include the excess benefit in his or her income.”

A

No. The highly compensated employees may be required to pay taxes on all or part of the reimbursements.

125
Q
Each of the following are requirements imposed by law on qualified tax-advantaged retirement plans EXCEPT:
A)Plan documentation.
B)Employee vesting.
C)Selective employee participation.
D)Employee communications.
A

Rationale
The correct answer is “C.” Broad employee participation, as opposed to selective participation, is a requirement of a tax-advantaged retirement plan. All of the others are requirements for “qualified” plans.

126
Q

A defined benefit pension plan

A

A defined benefit (DB) pension plan is a type of pension plan in which an employer/sponsor promises a specified pension payment, lump-sum or combination thereof on retirement that is predetermined by a formula based on the employee’s earnings history, tenure of service and age, rather than depending directly on individual investment returns.

Traditionally, many governmental and public entities, as well as a large number of corporations, provide defined benefit plans,

127
Q

safe harbor test

A

General rule (under Section 410(b)): The employer must cover at least 70% of all non-excludable, non-highly compensated employees. The general rule is commonly referred to as the safe harbor test.

Plans that do not meet the safe harbor test must satisfy one of the following two exceptions—the ratio percentage test or the average benefits percentage test.

128
Q

The ratio percentage test

A

the plan must cover a percentage of nonhighly compensated (NHC) employees that is at least 70% of the percentage of highly compensated (HC) employees

covered ratio percentage test⇒ %NHC covered / % HC covered ≥70%

129
Q

The average benefits percentage test

A

It has two components to satisfy annually

  1. ) The nondiscriminatory component
    a. ) Classification is reasonable and based on objective business criteria (e.g., job categories, location, hours, or weeks).
    b. ) The ratio percentage of the plan can be either ≥ 70% or be nondiscrimi-natory based on facts and circumstances.

2.) The average benefits percentage test—The average benefits percentage accrued for nonhighly compensated employees as a group must be ≥ 70% of the average benefits percentage accrued for the HC employees as a group.

average benefits percentage test ⇒ average benefits % NHC / average benefits % HC ≥70%

130
Q

What are advantages of profit sharing plans to businesses and business owners?

A

A)Allows discretionary contributions.
B)Must PERMIT withdrawal flexibility.
An advantage of profit sharing plans is that they PERMIT withdrawal flexibility.
C)Controls benefit costs.
D)May provide legal discrimination in favor of older owner-employees.

131
Q

Describe a defined contribution plan

A

the participant’s accrued benefit at any point is the participant’s present account balance. The accrual for the specific year is the amount contributed to the plan on the employee’s behalf for that year.

132
Q

Describe a defined benefit plan

A

the accrued benefit is the benefit earned to date, using current salary and years of service. The accrued benefit earned for the year is the additional benefit that has been earned based upon the current year’s salary and service.

133
Q

A negative election

A

A negative election is a provision whereby the employee is deemed to have elected a specific deferral unless the employee specifically elects out of such election in writing.

The IRS has approved a negative election scenario under which the compensation of newly hired employees, who do not affirmatively elect to receive cash or have a specified amount contributed to the employer’s 401(k) plan, is automatically reduced by three percent.

134
Q

an employer can not terminate a qualified retirement plan if

A

Retirement plans must not be created as a tax shelter for the owner. If they have been, plan termination can result in retroactive disqualification.

135
Q

All of the following statements concerning cash balance pension plans are correct EXCEPT:
A)The cash balance plan is generally motivated by two factors: selecting a benefit design that employees can more easily understand, and as a cost saving measure.
B)The cash balance plan is a defined benefit plan.
C)The cash balance plan has no guaranteed annual investment return to participants.
D)The cash balance plan is subject to minimum funding requirements.

A

Rationale

The correct answer is “C.” A basic component of a cash balance plan is the guaranteed minimum investment return.

136
Q

Which of the following statement(s) regarding 403(b) plans is true?

I. Assets within a 403(b) plan may be invested in individual securities.

II. A 403(b) plan usually provides a 3 to 7 year graduated vesting schedule.

III. A 403(b) plan must pass the ACP test if it is an ERISA plan.

IV. In certain situations, a participant of a 403(b) plan can defer an additional $9,500 as a catch up to the 403(b) plan.

A)IV only.
B)I and II only.
C)III and IV only.
D)II, III, and IV only.

A

Rationale
The correct answer is “C.” 403(b) plan assets cannot be invested in individual securities, and contributions to 403(b) accounts are always 100% vested. Statements “III” and “IV” are true. Remember, if an employee qualifies for the 15 year rule, the maximum elective deferral for 2020 may be as high as $29,000 ($19,500 deferral, plus $3,000 from the 15 year rule, plus $6,500 for the 50 and over catch-up).

A 403(b) plan, also known as a tax-sheltered annuity plan, is a retirement plan for certain employees of public schools, employees of certain Code Section 501(c)(3) tax-exempt organizations and certain ministers. A 403(b) plan allows employees to contribute some of their salary to the plan

137
Q

Mike and Lola are both 60 years old. They are married filing joint and have AGI of $150,000 (which is all comprised of earned income). Mike’s employer offers a 401(k) plan and although he is eligible to defer he does not make any deferrals into the plan. The employer allocates forfeitures to plan participants. This year, $5 of forfeitures were allocated to Mike’s account. Lola does not work outside the home and does not have any earned income. Which of the following is true?
A)They can both make deductible contributions to a Traditional IRA.
B)Neither of them can make deductible contributions to a Traditional IRA but they can both make a Roth IRA contribution.
C)Mike can make a contribution to a Roth IRA but Lola can’t.
D)Lola can make a deductible contribution to a Traditional IRA but Mike cannot.

A

Rationale
The correct answer is “D.” Mike is considered an active participant because a forfeiture allocation was made to the plan on his behalf. Therefore, he can contribute to a Traditional IRA but will be unable to deduct because he is above the AGI limitation for a MFJ active participant ($104,000 - $124,000) (2020).
He could contribute to a Roth IRA because he is below the AGI limitation of $196,000 - $206,000 (2020). Incidentally, because he is 50 or older he is allowed to make the $1,000 (2020) catch up contribution.

Lola is not an active participant because she is not covered by any plan. Although she doesn’t have any earned income she can utilize her husband’s earned income. Since she is not an active participant but her husband is, she will utilize the spousal phase out limitation of $196,000 - $206,000 (2020). Since their AGI is below this amount she can make a deductible contribution to a Traditional IRA. Alternatively, she could contribute to a Roth IRA because she is below the AGI limitation of $196,000 - $206,000 (2020). Incidentally, because she is 50 or older she is allowed to make the $1,000 (2020) catch up contribution.

138
Q
Satish has AGI of $66,000 (which is all comprised of earned income). She is single and age 51. Her employer made a $6,300 contribution to her SEP for the current year. What is her available deduction allowed for a Traditional IRA contribution?
A)$26,000
B)$6,300
C)$6,000
D)$7,000
A

Rationale
The correct answer is “B.” She can contribute to a Traditional IRA since she has earned income. She is considered an active participant because her employer made a contribution to the SEP on her behalf. Her deduction will be limited because she is within the AGI limitation for a single active participant ($65,000 - $75,000) (2020). She is also entitled to the catch up contribution of $1,000 because she is 50 or older. Therefore, her deductible contribution is: 7,000 x ((66,000 - 65,000)/10,000) = $700 is not allowed so $7,000 - $700 = $6,300 is permitted. Although the question didn’t ask - alternatively, she could contribute to a Roth IRA because she is below the AGI limitation of $124,000 - $139,000 (2020).

139
Q

Which of the following is/are a defined benefit plan formula(s)?
A)Unit benefit (a.k.a. percentage of earnings per year of service) formula.
B)Flat-percentage formula.
C)Flat-amount formula.
D)All of the above.

A

Rationale

The correct answer is “D.” All of the above are benefit formulas used by defined benefit plans.

140
Q
Sean, age 75 and Jaclyn, age 45, are married filing joint and have AGI of $200,000 (which is all comprised of earned income). Neither are active participants in a qualified plan. If they contributed the maximum allowed by law to their Traditional IRAs what is their available Above the Line Deduction for these contributions?
A)$0
B)$6,000
C)$12,000
D)$13,000
A

Rationale
The correct answer is “B.” Since they are not active participants there is no AGI limitation. However, Sean is above 70.5 and therefore is unable to make a contribution to a Traditional IRA. Jaclyn is able to make a $6,000 (2019) deductible contribution. She does not qualify for the catch up because she is not 50 or older.

141
Q
Which of the following clauses in a 401(k) plan can assist the plan in meeting the requirements of the ADP test?
A)Attestation clause.
B)No-Contest clause.
C)Negative election clause.
D)Deferral plan clause.
A

Rationale
The correct answer is “C.” A negative election clause can assist a 401(k) plan in meeting the ADP test because it automatically deems that an employee defers a specific amount unless he elects out of the automatic deferral amount. Options “A” and “D” do not exist and Option “B” is a clause commonly found in a will.

142
Q

Which of the following accurately describes a 403(b) plan?
A)A 403(b) plan is a noncontributory qualified profit sharing plan.
B)Because of catch-up provisions, the investment risk of the assets within a 403(b) plan is borne by the plan sponsor not the participant.
C)A participant’s benefits within a 403(b) plan will generally vest according to a 3 to 7 year graduated vesting schedule, however, a 5-year cliff vesting schedule may be used.
D)403(b) plan assets can be invested indirectly in stocks and bonds through annuities or mutual funds.

A

Rationale
The correct answer is “D.” Option “D” is a correct statement accurately describing a 403(b) plan. Option “A” is incorrect as a 403(b) plan is an employee deferral plan and is not a qualified plan. Option “B” is incorrect as the investment risk is borne by the employee in all cases. Option “C” is incorrect as an employee’s benefit within a 403(b) plan is always 100% vested.

143
Q

Which of the following legal requirements apply to Employee Stock Ownership Plans (ESOPs)?

I. ESOPs must permit participants, who are aged 55 or older and who have at least 10 years of service, the opportunity to diversify their accounts.

II. ESOPs can be integrated with Social Security.

III. An employer’s deduction for ESOP contributions and amounts made to repay interest on an ESOP’s debt cannot exceed 25% of the participant’s payroll.

IV. The mandatory 20% income tax withholding requirement does not apply to distributions of employer stock from an ESOP.

A)I and II only
B)II and IV only
C)I, II and III only
D)I and IV only

A

Rationale
The correct answer is “D.” Deductions for interest payments are not limited for ESOP plans. Deductions for repayment of principal is limited to 25% of covered compensation.

144
Q
Which of the following vesting schedules may a top-heavy qualified profit sharing plan use?
A)1 to 5 year graduated.
B)5-year cliff.
C)3 to 7 year graduated.
D)4 to 8 year graduated.
A

Rationale
The correct answer is “A.” As a result of the PPA 2006, qualified profit sharing plans must use a vesting schedule that provides participants with vested benefits at least as rapidly as either a 2 to 6 year graduated vesting schedule or a 3-year cliff vesting schedule. This requirement applies without regard to whether the profit sharing plan is a top-heavy plan. Options “B,” “C” and “D” all vest less rapidly than the required schedule.

145
Q

Which of the following vesting schedules may a non-top-heavy profit sharing plan use?

I. 2 to 6 year graduated.

II. 3-year cliff.

III. 1 to 4 year graduated.

IV. 3 to 7 year cliff.

A)I only
B)II and III only
C)I, II and III only
D)I, II, III and IV

A

Rationale
The correct answer is “C.” As a result of the PPA 2006, a profit sharing plan must vest at least as rapidly as a 3-year cliff or 2 to 6 year graduated schedule without regard to the plan’s top-heavy status. The profit sharing plan can follow any vesting schedule that provides a more generous vesting schedule.

146
Q

Deepak made a contribution to his Roth IRA on April 15, 2018 for 2017. He was 58 years of age at the time and decided it was time he made his first contribution to a Roth IRA. Over the years he has made $30,000 in contributions. On May 15, 2020 the entire account balance was $50,000 and he took out $45,000 to pay for his wedding and honeymoon. Which of the following statements is true?

A)He will not include anything in income and will not be subject to the 10% early withdrawal penalty.
B)He will include $15,000 in income and will be subject to the 10% early withdrawal penalty on $15,000.
C)He will include $15,000 in income but will not be subject to the 10% early withdrawal penalty.
D)He will include $20,000 in income but will not be subject to the 10% early withdrawal penalty.

A

Rationale
Solution: The correct answer is C.

Roth distributions are tax free if they are made after 5 years and because of

1) Death, 2) Disability, 3) 59.5 years of age, and 4) First time home purchase. He does meet a qualifying reason because he is over 59.5 in 2020 if he was 58 in 2018. However, he did not meet the 5 year holding period. He only has about 4.5 years. His distribution does not received tax free treatment. The treatment for a non-qualifying distribution allows the distributions to be made from basis first, then conversions, then earnings. His basis will be tax free, leaving only the earnings as taxable income. Since he did not take the entire account balance he will only be subject to tax on the $15,000 of earning withdrawn. The 10% penalty does not apply to this distribution since he qualifies for the 59.5 exception to the penalty.

147
Q

5215Question 3 of 25Retirement and EE Benefits Quiz 12

Kyle is 54 and would like to retire in 11 years. He would like to live the “high” life and would like to generate 90% of his current income. He currently makes $150,000 and expects $24,000 (in today’s dollars) in Social Security. Kyle is relatively conservative. He expects to make 8% on his investments, that inflation will be 4% and that he will live until 104. How much does Kyle need at retirement?
A)$3,631,802
B)$3,423,275
C)$3,554,911
D)$3,480,448
A
Rationale
The correct answer is "C." Salary = 111,000 (150,000* 90%) - 24,000 = 111,000
N = 11 years to retirement
I = 4% inflation
PV = 111,000 in salary
FV = 170,879.40

BEG PMT = 170,879.40
N = 39 104 - 65
I = 3.8462 Inflation adjusted rate of return = [(1.08/1.04) - 1] X 100
Solve for PV

Answer “A” is the wrong payment (150,000 -24,000* 90%) = 113,400.
Answer “B” is ordinary annuity (end mode).
Answer “D” uses 4% for interest (beg mode).

148
Q

Kevin, a 55-year-old corporate executive, wants advice as to when he can retire. His current salary is $240,000 and he receives an annual bonus of $300,000; he also has annual stock options and restricted stock awards valued at $100,000. His employer contributes to a cash balance pension plan and matches his contributions to a 401(k). Kevin owns a whole life insurance policy with a $500,000 death benefit and is considering the purchase of a term policy with a $2,000,000 death benefit. He and his wife, Anne, also age 55, believe they can live on an after-tax income of $180,000. Assume a federal income tax rate of 35%.

Kevin’s non-qualified stock options are as follows:
2,000 shares, strike price $34

5,000 shares, strike price $30

Current stock price: $65

Kevin’s tax bracket: 42% (federal and state)

Kevin has decided to exercise the above stock option awards which will expire in the next 2 years. Assuming he exercises them today, what is his tax liability (CFP® Certification Examination, released 8/2012)?

A)$35,550
B)$68,250
C)$91,660
D)$99,540

A

Rationale
Answer: D

Non-qualified stock options are taxed on the “bargain element” (difference between the market price and the strike price) as ordinary income when exercised. (Market Price – strike price) x Number of Shares x Tax Rate = Tax

Therefore, on the first NQ grant of 2,000 shares the tax is:

($65-34) x 2,000 shares = $62,000 in ordinary income. At 42% tax rate the tax is $62,000 x .42 = $26,040.00

And

On the second NQ grant of 5,000 shares the tax is:

($65-30) x 5,000 = $175,000 in ordinary income. At 42% tax rate the tax is $175,000 x .42 = $73,500.00

Total tax therefore is $26,040.00 + $73,500.00 = $99,540.00

149
Q

A rabbi trust is established to ?

A

A rabbi trust is established to avoid constructive receipt.

150
Q

Which of the following statements regarding ISOs and NQSOs is correct?
A)The income tax treatment of a cashless exercise of an ISO is favorable compared to the cashless exercise of a NQSO.
B)One of the disadvantages of an ISO is that the sale of the stock attributable to an ISO may result in the taxpayer paying alternative minimum tax.
C)IRC 409A provides harsh penalties when a company grants an ISO or NQSO with a strike price that exceeds the current FMV of the employer’s tax.
D)To the extent that the aggregate fair market value of stock with respect to which incentive stock options are exercisable for the 1st time by any individual during any calendar year exceeds $100,000, such options shall be treated as options which are not incentive stock options.

A

Rationale
The correct answer is d. Choice a is not correct as the tax treatment is the same for a cashless exercise of an ISO and an NQSO. Choice b is not correct. The sale of an ISO share of stock will generally have a negative adjustment for AMT, not positive, and therefore, it would not result in AMT. Choice c is not correct as 409A would apply if the strike price was less than the FMV on the date of grant.

151
Q

Jacque’s wife just lost her job and they had a death in the family. Jacque is planning on taking a hardship withdrawal from his 401(k) plan to pay for living expenses and funeral costs. Which of the following is correct regarding hardship withdrawals?
A)Hardship withdrawals can be taken even if there is another source of funds that the taxpayer could use to pay for the hardship.
B)Hardship withdrawals are beneficial because although they are taxable, they are not subject to the early withdrawal penalty.
C)Hardship withdrawals can be taken from elective deferral amounts or vested employer contributions.
D)Unless the employer has actual knowledge to the contrary, the employer may rely on the written representation of the employee to satisfy the need of heavy financial need.

A

Rationale
The correct answer is d. Answer a is not correct as there must not be another source of funds. Answer b is not correct as they are generally subject to a penalty unless there is an exception under IRC 72(t). Answer c is not correct as a hardship distribution can only be taken from employee deferrals.

152
Q
Maria, age 28, has just expressed an interest in retiring at age 55 and having an income of the equivalent of $40,000 per year in retirement income in today's dollars. She assumes that she can make 8% interest after tax and expects inflation to average about 4% per year. Her life expectancy is 85 years old and she wants to know how much she should be saving each year in her savings plan to reach her goal between now and her retirement.
A)$7,625
B)$24,159
C)$8,068
D)$15,311
A

Rationale
The correct answer is “B”. HP12C: Step #1 - NPV at time period zero: 0 g CFo, 0 g CFj, 26 g Nj, 40,000 g CFj, 30 g Nj, [(1.08/1.04) - 1] x 100 i, f NPV gives $264,184.34; Step #2 Annual savings required: N = 27, i = 8, PV = $264,184.34, PMT = ?, FV = 0, Answer: $24,159.

HP10BII: Step #1 - NPV at time period zero: 0 CFj, 0 CFj, 26 shift Nj, 40,000 CFj, 30 shift Nj, 1.08/1.04 = -1 = x 100 = i/yr, Shift NPV gives $264,184.34; Step #2 - Annual savings required: N = 27, I = 8, PV = $264,184.34, PMT = ?, FV = 0, Answer: $24,159.

153
Q
Bank Corp has a defined benefit plan with 60 employees. What is the minimum number of employees the defined benefit plan must cover to conform with the requirements set forth by the IRC?
A)24
B)30
C)42
D)50
A

Rationale
The correct answer is “A.” The plan must cover the lesser of 50 people or 40% of all employees. In this case, the lesser would be 40% of 60, or 24 people.

154
Q

You are in the process of advising your business client with regard to a non-qualified stock option plan that he is considering newly instituting as a program in his business for his employees. Before beginning, which of the following are questions that must be addressed as essential and pertinent to the stock option issue?

I. What is the earliest date you can exercise the option?

II. What do you need to do when you exercise the option?

III. Can you exercise using stock you own?

IV. When will the option terminate?

V. Can you exercise after your employment terminates?

A)I, III and V only.
B)II and IV only.
C)IV only.
D)All of the above.

A

Rationale
The correct answer is “D.” All of these questions must be addressed before one undertakes any type of Stock Option plan for employees.

155
Q

What are differences between the safe harbor 401(k) plan and a 401(k) plan with a qualified automatic contribution arrangement?

A

The safe harbor 401(k) plan has more liberal (better for employees) vesting for employer matching contributions as compared to 401(k) plans with a qualified automatic contribution arrangement.

Safe harbor plans require 100% vesting, while 401(k) plans with QACAs require two year 100% vesting. The matching contributions are different for the plans. Employees are not required to participate in either plan. Both plans eliminate the need for ADP testing, which means that they eliminate the need for qualified matching contributions and corrective distributions.

156
Q

What are the difference between the treatment of ‘qualified’ versus ‘non-qualified’ stock options?

A

If an employee receiving incentive stock options does not meet the employment time requirement, but receives options as a non-qualifying and exercises them, The employee will be required to recognize compensation income in the year the option is exercised.

This illustrates the difference between the treatment of ‘qualified’ versus ‘non-qualified’ stock options. The tax implications are immediate and the income is recognized as soon as the option is exercised rather than when the stock is subsequently sold.

157
Q

83(b) election for restricted stock

A

The 83(b) election is a provision under the Internal Revenue Code (IRC) that gives an employee, or startup founder, the option to pay taxes on the total fair market value of restricted stock at the time of granting.

It’s a letter you send to the Internal Revenue Service letting them know you’d like to be taxed on your equity, such as shares of restricted stock, on the date the equity was granted to you rather than on the date the equity vests.

158
Q

Are Roth distributions tax free?

A

Roth distributions are tax free if they are made after 5 years and because of 1)Death, 2)Disability, 3) 59.5 years of age, or 4)First time home purchase. Although he met the 5 year rule, he did not meet one of the four qualifying reasons.
His distribution does not received tax free treatment. The treatment for a non-qualifying distribution allows the distributions to be made from contributions first, then conversions, then earnings. In this case the distinction in distribution order is irrelevant since he withdrew the entire account balance. However, his contribution will be tax free, leaving only the $25,000 in earnings as taxable income. The 10% penalty does not apply to this distribution since he qualifies for the higher education exception to the penalty.

159
Q

Which plans are not qualified retirement plans?

A

1) Rabbi trust
2) Nonqualified stock option plan
3) ESPP

160
Q

For favorable tax treatment the option must be held two years and the stock for one year after exercise.

A

No taxable income will be recognized by the employee when the qualified option is granted or exercised.

161
Q

Ernest converted his Traditional IRA to a Roth IRA on Dec 15, 2016. He was 35 years of age at the time and had never made a contribution to a Roth IRA. The conversion was in the amount of $60,000 ($10,000 of contributions and $50,000 of earnings). Over the years he has also made $15,000 in contributions. On May 15, 2020 he withdrew the entire account balance of $100,000 to pay for a 1 year trip around the world. Which of the following statements is true?
A)$25,000 of the distribution will be subject to income tax and $85,000 of the distribution will be subject to the 10% early withdrawal penalty.
B)$25,000 of the distribution will be subject to income tax and the 10% early withdrawal penalty.
C)Some of the distribution will be taxable but the entire distribution will be subject to the 10% early withdrawal penalty.
D)None of the distribution will be taxable nor will it be subject to the 10% early withdrawal penalty.

A

Rationale
Solution: The correct answer is A.

Roth distributions are tax free if they are made after 5 years and because of 1)Death, 2)Disability, 3) 59.5 years of age, and 4)First time home purchase. He does not meet the five year holding period or one of the exceptions. His distribution does not received tax free treatment. The treatment for a non-qualifying distribution allows the distributions to be made from basis first, then conversions, then earnings. His basis will be tax free. The conversion is also tax free since we paid tax at the time of the conversion on those earnings. The remaining earnings since establishment of the Roth are $25,000 (100,000 - $15,000 in basis - $60,000 in conversions) and will be taxed. The 10% penalty does apply to this distribution since he does not qualify for any of the exceptions to the penalty. The contributions escapes penalty but the conversions and earnings of $85,000 are subject to the 10% early withdrawal penalty. Remember that in order for the conversions to escape the 10% early withdrawal penalty the distribution must occur after a 5 year holding period beginning Jan 1 in the year of conversion or meet one of the 10% early withdrawal exceptions.

Summary:

$60,000 paid tax at conversion. Subject to penalty

$15,000 in contributions no tax, no penalty

$25,000 earnings. Taxable and subject to penalty

162
Q

Randal was just hired by Chastain, Inc., which sponsors a defined benefit plan. After speaking with the benefits coordinator, Randal is still confused regarding eligibility and coverage for the plan. Which of the following is correct?
A)The plan could provide that employees be age 26 and have 1 year of service before becoming eligible if upon entering the plan, the employee is fully (100%) vested.
B)The plan may not cover Randal due to his position in the company, even if Randal meets the eligibility requirements.
C)Part-time employees, those that work less than 1,000 hours within a twelve-month period, are always excluded from defined benefit plans.
D)Generally, employees begin accruing benefits as soon as they meet the eligibility requirements.

A

Rationale
The correct answer is b. Choice a is not correct because the general eligibility is age 21, not 26. Choice c is not correct because a plan could cover part time employees, but will generally not. Choice d is not correct because employees become part of a plan only as early as at the next available entrance date after meeting the eligibility requirements.

163
Q

Eldrick, age 40, established a Roth IRA 3 years ago and was tragically struck and killed by an errant golf ball hit by a drunk spectator at a golf tournament. Eldrick had contributed a total of $10,000 to the account and had converted $20,000 from his traditional IRA. His 20 year old son, Charlie, inherited the Roth IRA, which now has a balance of $60,000. Which of the following statements is correct?
A)Charlie can distribute the entire balance from the Roth IRA without it being subject to any income tax or penalty the month after Eldrick dies.
B)Charlie must take minimum distributions from the Roth IRA the year Eldrick dies.
C)If Charlie begins taking minimum distributions, then the first distribution will be partially taxable.
D)Charlie could delay taking a distribution from the Roth IRA for several years and avoid all penalties and income tax on the distribution.

A

Rationale
The correct answer is d. Answer a is incorrect because the distribution would not be a qualified distribution since the five year rule has not been met. Answer b is not correct as he could begin taking distribution the year following death or take a full distribution within five years. Answer c is not correct, because the first distribution would consist of previously taxed contributions and would therefore not be taxable. Answer d is correct as he could delay taking a distribution from the account for two years. At that point, the distribution would be qualified since it meets the five-year rule and is on account of death. The distribution would avoid all income tax and penalties.

164
Q
Lien, age 35, recently left his employer, GoGoRoller, a roller blade manufacturer. He left after 18 months because the working conditions were unbearable. GoGoRoller sponsored a SIMPLE IRA. Lien deferred $3,000 into the plan during his time there and the employer contributed $1,500. When he terminated he withdrew the entire account balance of $4,750. Assuming he is in the 15% tax bracket, what is the tax and penalty consequence for this distribution?
A)$712.50
B)$775.00
C)$1,187.50
D)$1,900.00
A

Rationale
The correct answer is “D.” SIMPLE IRAs require a 25% penalty for early withdraws in the first two years if the participant does not meet any of the early withdrawal exceptions. He does not meet any of the exceptions and the distribution is within the first two years. The breakdown of employee deferrals, employer contributions and earnings is irrelevant. Therefore, his tax and penalty consequence is $1,900 = $4,750 x 40%. The 40% is represented by 15% tax plus 25% penalty.

165
Q

Qualified Pre-Retirement Survivor Annuity (QPSA)

A

A qualified pre-retirement survivor annuity (QPSA) is a death benefit that is paid to the surviving spouse of a deceased employee.
If the employee dies before retirement, the qualified pre-retirement survivor annuity is paid to offer compensation to the surviving spouse for the loss of retirement benefits that would have otherwise been paid to the employee. As the name implies, QPSAs are paid only in the case of qualified plans.

166
Q

waiver

A

n. 放弃;弃权;弃权证书

167
Q

Which of the following is/are elements of an effective waiver for a preretirement survivor annuity?

I. The waiver must be signed within six months of death.

II. The waiver must be signed only by a plan participant.

III. The waiver must be notarized or signed by a plan official.

A)III only
B)I and II only
C)II and III only
D)I, II and III

A

Rationale
The correct answer is “A.” Both the plan participant and the nonparticipant spouse must sign the waiver. Statement “III” is correct. Statement “I” is unfounded.

168
Q

Which of the following statements concerning the use of life insurance as an incidental benefit provided by a qualified retirement plan is (are) correct?

I. The premiums paid for the life insurance policy within the qualified plan will trigger a taxable event for the participant at the time of payment.

II. Under the 25 percent test, if term insurance or universal life is involved, the aggregate premiums paid for the policy cannot exceed 25 percent of the employer’s aggregate contributions to the participant’s account. If a whole life policy other than universal life is used, however, the aggregate premiums paid for the whole life policy cannot exceed 50 percent of the employer’s aggregate contributions to the participant’s account. In either case, the entire value of the life insurance contract must be converted into cash or periodic income at or before retirement.

A)I.
B)II.
C)I and II.
D)Neither I or II.

A

Rationale
The correct answer is “C.” Every year the plan participant pays income tax on the dollar value of the actual insurance protection – approximately equal to the term insurance cost. This is commonly called the PS58 cost. The sum of all those costs is the participant’s basis.

169
Q

Joseph wants to take a loan from his 403(b) plan. Which of the following requirements will apply to his situation?
A)The loan can exceed $50,000 only if the account is worth more than $100,000.
B)The loan must be paid back with quarterly payments over three years, unless used to purchase a principal residence.
C)The loan is exempt from interest charges since the participant is borrowing his own money.
D)The plan document may provide for a maximum and minimum loan amount.

A

Rationale
The correct answer is “D.” Maximum loan amount is lesser of 50% of vested amount or $50,000 paid in quarterly (or more frequent) payments over five years, unless used for home purchase. Loan must carry reasonable interest rate.

170
Q

Your Uncle Ben began receiving required minimum distributions from his IRA in 2005 and has died leaving a balance in his IRA. He has named you as beneficiary (non-spouse beneficiary). Which of the following identifies your minimum distribution rule?

A)Distributions may be made over your life expectancy (fixed) beginning by December 31st in the year of death.
B)Distributions may only be made over a five-year period.
C)Distributions must be made over Uncle Ben’s life expectancy (recalculated) with the first distribution made by December 31st of the year following death.
D)Distributions may be made over your life expectancy (fixed) beginning by December 31st of the year following death.

A

Rationale
The correct answer is “D.” Note that for a non-spouse beneficiary, the Single Life Table is entered once in the year following death based on the age of the beneficiary and then that number declines by a factor of 1 each year to determine the divisor for the following distributions. The non-spouse beneficiary could make a direct transfer to an “inherited IRA”. Answer “A” is incorrect because the distributions to the beneficiary are based on the beneficiary’s life expectancy beginning in the year following death. Do point out that the distribution in the year of death is still based on the owner’s life. Answer “B” is incorrect because the five-year rule is NOT the only choice. Answer “C” is incorrect because a non-spouse beneficiary would not receive distributions based on the owner’s life expectancy nor would a post-death non-spouse distribution be recalculated.

171
Q
A client became age 70 1/2 on October 1, 2019 and must receive a minimum distribution from his IRA account. The value of the account at the beginning of the current year was $53,000. His spouse, age 63, is the beneficiary of the IRA account. The applicable divisor for his distribution is 26.5. If the client takes a $1,000 distribution by April 1, what is the tax penalty, if any?
A)$0
B)$100
C)$150
D)$500
A

Rationale
The correct answer is “D.” The minimum distribution is calculated by taking the account value at the beginning of the year ($53,000 in this case) and dividing by the applicable divisor (life expectancy 26.5 years in this example). The resulting figure is the minimum distribution ($2,000). Because the amount distributed was $1,000 less than required under minimum distribution rules, a 50% penalty applies. 50% of the undistributed minimum distribution is $500.

Note: It was age 70½ before the passage of the Setting Up Every Community for Retirement Enhancement (SECURE) Act in December 2019.1

 Anyone who is covered by the old rules (those born before July 1, 1949) has already begun paying RMDs and must continue to do so. Everyone else can wait until April 1 of the year following the year in which they reach age 72.

172
Q
Marilyn Hayward is the sole proprietor and has no employees in her unincorporated Graphics for Green Promotions. In 2020, Marilyn established a profit sharing Keogh plan with a 25% contribution formula. As of December 31, 2020, Marilyn has $140,000 of Schedule C net earnings. The deduction for one-half of the self-employment tax is, therefore, $9,891. What is the maximum allowable Keogh contribution that Marilyn can make?
A)$19,500
B)$24,311
C)$26,022
D)$32,560
A

Rationale
The correct answer is “C.” Half of the self-employment tax is given, you can skip the 1st step in the self-employment contribution formula (multiplying by 92.35%)

140,000 - $9,891 = 130,109 x 20%*= 26,021.80

*The 20% is from the contribution rate formula: contribution rate / (1 + contribution rate) = self-employment contribution rate
.25/ (1+.25) = .20. Note that if the employer has employees and contributes 15% for them, that is the contribution rate to use in this formula.

173
Q

Carolyn Smart wanted to volunteer full-time and decided to retire from Lotsa Cash Corporation at the age 57, after 15 years of service. She requested a total distribution of her account in the Lotsa Cash Corporation’s profit sharing plan and received a check, made payable to her. Her account balance was $60,000 on her final day of employment. Which of the following statements describe the consequences of this distribution?

I. Eligible for 10 year forward averaging

II. Subject to 10% penalty

III. Eligible for Rollover

IV. Subject to mandatory 20% withholding

V. Exempt from the 10% early withdrawal penalty

A)I, II and III only
B)II, III and IV only
C)III, IV and V only
D)III and IV only

A

Rationale
The correct answer is “C.” Statements II and V cannot co-exist. She is not old enough for statement I. She is not subject to 10% penalty because this is a qualified plan and she is over 55. Distributions from qualified plans are subject to mandatory 20% withholding.

174
Q

What are basic characteristics of non-qualified deferred compensation plans is accurate?

A

Employer contributions into a non-qualified plan are not tax deductible to the employer until the employee has constructive receipt and is taxed on the income. Most non-qualified plans are designed to avoid constructive receipt by the employee until retirement.

In an informally funded plan (Rabbi trust), the employee has the segregated assets as security of the agreement, assuming the employer remains solvent and the assets are not taken by the employer’s creditors. This risk of having creditors take the assets inside a “Rabbi trust” is what constitutes a substantial risk of loss or forfeiture and keeps the employee from being considered in “constructive receipt” of the formally funded assets.

175
Q

Which of the following statements is/are characteristic(s) of Tax-Sheltered annuities (TSAs)?

I. Salary reduction contributions are NOT reported as W-2 income and are subject to Social Security tax.

II. The maximum salary deferral limit is $19,500 for a newly hired employee in 2020, under age 50.

III. Employer contributions are deductible by the employer.

IV. Loans and “catch-up” contributions may be permitted.

A)IV only.
B)I and III only.
C)I, II and III only.
D)I, II, III and IV.

A

Rationale
The correct answer is “D.” TSA salary deductions are subject to Social Security (payroll) taxes; contributions by the employer to the TSA are tax deductible by the employer but these contributions are not subject to payroll taxes, much like 401K contributions. Remember, TSAs are also known as a 403(b) retirement plan.

176
Q

Which of the following is NOT true concerning eligibility or “waiting periods”?
A)A qualified plan can require a waiting period of up to two years.
B)In order to have graded vesting, the eligibility period must be no longer than 12 months.
C)Any plan which has an eligibility period exceeding 12 months must have immediate vesting.
D)The eligibility period can vary between classes of employees.

A

Rationale
The correct answer is “D.” The eligibility requirements must be the same for all employees. All other statements are accurate.

177
Q

off-shore

A

adj. 离开海岸的;国外的

178
Q

sources of statutory law concerning qualified retirement plans:

A

The IRC and ERISA are laws or statutes passed by Congress.
The Labor Department and Private Letter rulings are sources of regulatory law. Of course, the U.S. Tax Court only interprets statutory and regulatory law (theoretically), and is not a source of laws.

179
Q

One of the disadvantages of an ESOP is that the stock is in an undiversified investment portfolio. Which of the following statements is correct about ESOPs?

I. An employee, age 55 or older, who has completed 10 years of participation in an ESOP may require that 25 percent of the account balance be diversified.

II. An employee who receives corporate stock as a distribution from an ESOP may enjoy net unrealized appreciation treatment at the time of distribution.

A)I only.
B)II only.
C)I and II.
D)Neither I or II.

A

Rationale
The correct answer is “C.” Both statements are correct. ESOP distribution in stocks are NUA (Net Unrealized Appreciation). Employees in an ESOP may demand 25% of the current balance be diversified.

180
Q
Robert Sullivan, age 56, works for Dynex Corporation, and earns $290,000. Dynex Corp. provides a non-elective 2% contribution to its SIMPLE IRA plan. Which one of the following is the maximum amount that could go into Robert’s account this year?
A)$17,200
B)$19,500
C)$22,200
D)$22,300
A

Rationale
The correct answer is “C.” The compensation limit of $285,000 applies to SIMPLE IRAs when non-elective contributions are made. Therefore the employer contribution is $5,700 and the employee can contribute up to $13,500 for 2020. In addition, Robert is 50 years old or older so he may make an additional catch-up contribution of $3,000. His total contribution is $5,700 + $13,500 + $3,000 = $22,200.

181
Q

Kent Reeder, age 52, works as the administrator and curator at the Museum of Antique Manuscripts, a not-for-profit organization in Metropolitan Center. He has worked there 18 years and began contributing to the 403(b) plan 12 years ago but skipped contributing last year. He earns $85,000 a year. He has asked you to maximize his contribution. Which of the following is/are TRUE?

I. He may contribute $19,500 plus $6,500 for age 50+ catch-up, plus $3,000 long service catch-up.
II. He may not contribute to the long-service catch-up this year due to omitting a contribution last year.
III. He may contribute $19,500 plus $6,500 age 50+ catch-up.
IV. He may not participate in both the long service catch-up and the age 50+ catchup the same year.
V. He is not eligible for the long service catch-up.

A)III and V only.
B)II only.
C)I, III and IV only.
D)I and III only.

A

Rationale
Solution: The correct answer is A.

He is not eligible for the long service catch-up because the museum is not a Health, Education, Religious (HER) organization. The maximum contribution limits for 2020 are $19,500 plus the age 50+ catch-up of $6,500.

182
Q

Which analysis method can mitigate the risk of outliving retirement funds?

A

Capital Preservation Model

The Capital Preservation Model assumes at life expectancy, as estimated in the annuity method, the client has exactly the same account balance as he/she started with at retirement. So if life expectancy is exceeded there is still capital available.

183
Q
Donald and Daisy are married and file jointly. They are both age 42, both work, and their combined AGI is $115,000. This year Donald's profit sharing account earned over $5,000. Neither he nor the company made any contributions and there were no forfeitures. Daisy declined to participate in her company's defined benefit plan because she wants to contribute to and manage her own retirement money. (Her benefit at age 65 under the plan is $240 a month.) How much of their $12,000 IRA contribution can they deduct? Assume that $6,000 is contributed to each account.
A)$6,000
B)$8,700
C)$9,600
D)$12,000
A

Rationale
The correct answer is “B.” Daisy is an active participant. She cannot opt out of a defined benefit plan.

Reduction = 6,000 X [(115,000-104,000) ÷ 20,000]

Reduction = 3,300

$6,000 - $3,300 = $2,700 deductible contribution.

Donald is not active in the current year so he is eligible for a spousal IRA of $6,000.

$6,000 + $2,700= $8,700

Formula: Contribution x [(your AGI – the bottom of the phase out range) divided by the amount of the range (124,000-104,000 is where the 20k comes from)]

184
Q

The SECURE Act of 2019 changed RMD age to 72 for those turning 70 1/2 after 12/31/19.

A

His first RMD distribution was for 2019, the year he turned 70 1/2. His 2020 distribution is based on the $48,000 year end balance from 2019 and is subject to a 26.5-year life expectancy of the client, thus minimum distribution is $1,811 for 2020. If the client takes only $1,000, the balance ($811) is subject to a 50% excise tax or $411 penalty.

The SECURE Act of 2019 changed RMD age to 72 for those turning 70 1/2 after 12/31/19.

185
Q

Complex Corporation is ready to adopt a profit sharing plan for eligible employees. Which of the following groups would have to be considered in meeting the statutory coverage and participation tests?

I. Employees of Simple Corporation, in which Complex owns 85% of the stock.

II. Employees of Universal Corporation, in which Complex owns 55% of the stock.

III. Rank and file workers at Complex who are union members with a contract that provides retirement benefits as a result of good-faith collective bargaining.

IV. Employees who are leased and covered by the leasing corporation’s profit sharing plan.

A)I only
B)I and II
C)I and IV
D)II and III

A

Rationale
The correct answer is “C.” Simple must be considered because Complex owns more than 80% and the leased employees must be considered because their leasing company’s is not a pension plan. Universal would not be considered a subsidiary because it is only 55% not more than 80%. The union employees are excluded from testing by the IRC.

186
Q

characteristics of a non-qualified deferred compensation agreement for an individual?

A

I. It may provide for benefits in excess of qualified plan limits.

II. The contribution underlying the agreement may be structured as additional compensation to the employee.

III. It must be entered into prior to the rendering of services to achieve deferral of compensation.

IV. The contribution underlying the agreement may be paid from the current compensation of the employee.

The underlying contribution may be structured as additional compensation (a so-called salary continuation plan.) Statement “IV” represents a so-called pure deferred compensation plan.

187
Q

hich of the following statements are true in regards to Section 457 plans?

I. Eligible plan sponsors include non-profit organizations, churches, and governmental entities.

II. In-service distributions after age 59 1/2 are allowed in a 457 plan.

III. Salary deferrals are subject to Social Security, Medicare, and Federal unemployment tax in the year of the deferral.

IV. Assets of the plans for non-government entities are subject to the claims of the sponsor’s general creditors.

A)I and III only.
B)III and IV only.
C)I, II and IV only.
D)II, III and IV only.

A

Rationale
The correct answer is “B.” Churches are not qualifying sponsors of 457 plans. In-service distributions are not allowed until age 70 1/2.

188
Q

a profit-sharing plan

A

Retirement benefits can be adjusted based on sponsor profits, but the defined benefit plan can not adjust based on sponsor profits.

189
Q

defined benefit pension plans

A

An actuary is needed to calculate the minimum funding level.

The benefits in most traditional defined benefit plans are protected, within certain limitations, by federal insurance provided through the Pension Benefit Guaranty Corporation (PBGC).

190
Q

What are the tax ramifications of a non-qualified deferred compensation plan?

A

A participant in an unfunded plan will not be currently taxed if the promise of benefits is unsecured and the agreement is executed prior to the first day of service under the agreement.

Any funded plan is taxable unless a Rabbi trust is utilized.

the NPV of the guaranteed future income will be included in the gross estate of the deceased participant.

191
Q

Which of the following will be subject to a 10% early withdrawal penalty?

A)Sylvia, age 56, retired from Marshall Corporation. She takes a $125,000 distribution from the Marshall Corporation Defined Contribution Retirement Plan to pay for living expenses until she is eligible for Social Security.
B)Terry quits Shoe Shine Company at age 48. He begins taking equal distributions over his life expectancy from his qualified plan after separating from service. The annual distribution is $2,000.
C)Kevin’s wife had a baby 10 days ago. He withdrew $2,000 from his IRA to cover costs for baby furniture, diapers and carseats.
D)Edward, age 40, takes a $40,000 distribution from his profit-sharing plan to pay for his son’s college tuition.

A

Rationale
The correct answer is “D.”

There is no provision for a distribution without penalty under this circumstance. Edward is only 40 and education withdrawals are allowed in IRAs, not from qualified plans.

SECURE Act 2019 added penalty free withdrawals up to $5,000 taken within 12 months of birth or legal adoption.

192
Q

When calculating the Wage Replacement Ratio (WRR), what percentage of income is subtracted for a self-employed individual for Social Security and Medicare Taxes?

A

15.3%

193
Q

VEBA

A

A voluntary employees’ beneficiary association (VEBA) plan is a type of tax-exempt trust used by its members and eligible dependents to pay for eligible medical expenses.

It permits:

I. Life, sickness and accident benefits

I. Severance and supplemental unemployment

III. Job training

194
Q

What qualified retirement plans are subject to mandatory minimum funding requirements?

A

I. Defined benefit pension plans.

II. Money purchase pension plans.

III. Target benefit plans.

Profit sharing plans (including 401(k) plans) are exempt from minimum funding requirements, as are SIMPLE IRA plans.
All pension plans are subject to minimum funding requirements.

195
Q
Match the following statement with the type of retirement plan which it most completely describes: "A plan which requires annual employer contributions equal to a formula determined by each participant's salary" is a...
A)Profit sharing plan.
B)Money purchase plan.
C)SIMPLE IRA.
D)Defined benefit plan.
A

The correct answer is “B.” Defined benefit plan and cash balance plan (Answer “D”) contributions are determined by age, as well as salary. Answer “A” doesn’t require annual contributions. Answer “C” has employer contributions determined by the amount of employee deferrals.

196
Q

Bertha, who is 54 years old, spent most of her career in the corporate world and now provides consulting services and serves as a director for several public companies. Her total self-employment income is $500,000. She is not a participant in any other retirement plan today. She would like to shelter as much of her self-employment earnings as possible by contributing it to a retirement plan. Which plan would you recommend?
A)Establish a 401(k) plan.
B)Establish a target benefit plan.
C)Establish a Deferred Comp program for himself.
D)Establish a SEP.

A

Rationale
The correct answer is A. She will be able to defer $57,000 plus the catch up of $6,500 to the 401(k) plan, where she can only contribute $57,000 to the target benefit plan and the SEP. She cannot set up a deferred compensation plan and defer tax.

197
Q

A Supplemental Executive Retirement Plan (SERP)

A

A supplemental deferred compensation plan providing retirement benefits above the company’s qualified plan AND without regard to Section 415 limits.

SERP supplements the pension plan without regard to limits imposed upon salary levels (i.e., maximum salary of $285,000 in 2020) or the maximum funding levels of Section 415. Do not confuse with an excess benefit plan which extends the benefits of a company’s qualified plan above the Section 415 limits but still adheres to maximum salary limitations.

198
Q

cash balance plan

A

A Cash Balance Plan is a defined benefit plan that acts in some ways like a defined contribution plan. Because it incorporates elements of both, it is sometimes called a “hybrid” plan. A defined benefit plan provides a specific benefit at retirement for each eligible employee.

199
Q

Mike’s Mega Muffelettas (MMM) is a fairly large company based in Louisiana, with over 300 employees. MMM sponsors a defined benefit plan. George has worked at the company for the last 30 years and is looking forward to his retirement in another ten years. However, he just received a letter from the company that informs him that his defined benefit plan is being converted to a cash balance plan. What advice can you give George?
A)His benefit could freeze as a result of the conversion; a stituation known as “wearaway.”
B)The present value of the accrued benefit from the defined benefit should be preserved in the conversion and you should earn additional benefits under the cash balance plan.
C)The cash balance plan provides a guaranteed rate of return and benefits that are fully insured by the PBGC.
D)The cash balance plan acts like a defined contribution plan and will permit Geoge to self-direct his retirement assets.

A

Rationale
Answer b is correct because his benefits under the defined benefit plan must be preserved. Answer a is not correct as “wearaway” was done away with as it negatively effected employees who were near retirement and resulting in employees not accruing additional benefits after a conversion from a defined benefit plan to a cash balance plan. Answer c is not correct as the PBGC does not fully guarantee benefits. Answer d is not correct as a cash balance plan will have a guaranteed rate of return. Employees do not self-direct their assets in a cash balance plan.

200
Q

Which of the following is NOT included as one of the provisions for the continuation benefits under the Consolidated Omnibus Budget Reconciliation Act (COBRA) of 1985?
A)All employers offering group health insurance must provide COBRA continuation benefits.
B)COBRA benefit durations vary from 18, 29, or 36 months depending upon the qualifying event.
C)COBRA eligible coverages do not include: dental plans, vision plans, Medical FSAs, prescription drug plans and mental health plans.
D)COBRA eligibility ceases when a covered participant becomes eligible to participate in another group health plan or Medicare.

A

Rationale
The correct answer is “A.” COBRA benefits are required of employers who have 20 or more employees. Employers with fewer employees are not required to provide COBRA continuation, even if they offer a group a health insurance plan to employees.

201
Q

safe-harbor leasing rules

A

A lease to transfer tax benefits of ownership (depreciation and debt tax shield) from the lessee, if the lessee could not use them, to a lessor that could use them.

202
Q

Safe harbor requirements to exclude leased employees from an employer’s retirement plan include all but the following:
A)The leasing company must maintain a money-purchase plan with a contribution rate of 10%.
B)The retirement plan of the leasing company may be integrated.
C)The leasing company’s plan must provide immediate vesting.
D)Safe harbor can be used until leased employees constitute 20% of the non-highly compensated work force.

A

Rationale
The correct answer is “B.” Under the safe-harbor leasing rules the plan must provide a 10%, non-integrated money purchase plan with immediate vesting. No more than 20% of the employer’s non-highly compensated employees may be leased to qualify for the safe harbor rules.

203
Q

What factors increase funding costs of a Defined Benefit plan?

I. Low turnover rate.

II. Early retirement.

III. High interest rate.

IV. Late retirement.

V. Salary scale assumption.

A)I and III only.
B)I, II and V only.
C)I and V only.
D)II, III and V.

A

Rationale

The correct answer is “B.” Options I, II and V increase funding costs of a DB plan.

204
Q

David is awarded an immediately vested, non-qualified stock option for 1,000 shares of company stock with an exercise price of $35 per share while the stock price is currently $33 per share. What are the tax ramifications, if any at the date of the grant?

A

A)$0
Rationale
The correct answer is “A.” In the case of NQS Options, the option is not taxed at the grant if the exercise price is equal to or greater than the fair market value of the stock.

205
Q

According to ERISA, which of the following is/are required to be distributed automatically to defined benefit plan participants or beneficiaries?

I. Annual accrued benefit as of the end of the previous year.

II. The plan’s summary annual report.

III. A detailed descriptive list of investments in the plan’s fund.

IV. Terminating employee’s benefit statement.

A)I, II and III only.
B)I and II only.
C)II and IV only.
D)IV only.

A

Rationale
The correct answer is “C.” Employee accrued benefits are established by the pension formula, therefore are not required to be provided each year. (Defined contribution plans must provide an annual statement of account.) Because the participant has no individual account in a DB plan, a detailed list of investments is not required. The participant in a DB plan generally has no input in the investment choices within the plan.

206
Q

Under what condition, the penalty can be waived of withdrawing from 401K?

A

The IRS dictates you can withdraw funds from your 401(k) account without penalty only after you reach age 59½, become permanently disabled, or are otherwise unable to work.

207
Q

Your client’s only employer has established a payroll deduction TSA. Your client is single, making more than $61,000 per year. Which of the following is TRUE concerning the plan?
A)The phase-out maximum AGI of $60,000 in the current year does apply.
B)TSA contributions are not subject to Social Security taxes.
C)The employer usually controls the investment selections.
D)Contributions are not subject to Federal/State Withholding tax.

A

Rationale
The correct answer is “D.” Contributions to a TSA are made before Federal/State withholding taxes are applied. Phase-outs apply to traditional IRAs, not TSA plans. Salary reductions into a TSA are subject to Social Security and Medicare taxes. Employers usually allow the employee to control the allocation of assets within their TSA.

208
Q

In order to be qualified, money purchase plans must contain which of the following?

I. A definite and non-discretionary employer contribution formula.

II. Forfeitures can be reallocated to the remaining participants’ accounts in a non-discriminatory manner or used to reduce employer contributions.

III. An individual account must be maintained for each employee of employer contributions.

IV. The normal retirement age must be specified.

A)I and II only.
B)II and IV only.
C)I, II and III only.
D)II, III and IV only.

A

Rationale
The correct answer is “C.” A normal retirement age must be stated in a defined benefit plan, so Statement “IV” is incorrect. Defined contributions plans (such as a money purchase plan) have retirement benefits which are determined by the value of the individual account whenever the participant retires. Forfeitures may be allocated to employees’ individual accounts or used to reduce employer required contributions.

209
Q
WestN, Inc. sponsors a 401(k) profit sharing plan with a 50% match. In the current year, the company contributed 20% of each employee’s compensation to the profit sharing plan in addition to the match to the 401(k) plan. The company also allocated a forfeiture allocation of $4,000. The ADP of the 401(k) plan for the NHC is 4%. Wade, who is age 45, earns $190,000 and owns 19% of the company stock. If Wade wants to maximize the contributions to the plan, how much will he defer into the 401(k) plan?
A)$19,500
B)$11,400
C)$10,000
D)$5,000
A

Rationale
The correct answer is C. Wade is highly compensated because he is more than a 5% owner, so the maximum that he can defer to satisfy the ADP Test requirements is 6% (4% + 2%). Wade is also limited by the 415(c) limit of $57,000. Since the company contributes $42,000 (20% of $190,000 + $4,000 of forfeiture allocations), he only has $15,000 to split between the deferral and the match. Thus, he contributes $10,000* and the match is $5,000, which when added to the $42,000 totals $57,000. 6% of his salary of $190,000 is $11,400. However, he cannot defer this amount due to the 415(c) limit.

*When he contributes they match 50%, so for every dollar he contributes 1.5 x that amount goes into the plan.
Take 15,000/1.5 = $10,000

210
Q

Which statements below accurately reflect characteristics of the Tax Sheltered Annuity (TSA)?

I. Annuity payments from a TSA are taxed using the three-year rule.

II. Employers may make matching contributions or contribute a fixed percentage.

III. An employee under age 50, who contributed $8,000 to a 401(k) plan is limited to contributing a maximum of $11,500 to a salary reduction TSA.

IV. At the TSA owner’s death, the full amount of proceeds paid to beneficiaries is included in the gross estate of the decedent.

A)I, II and III only.
B)I, II and IV only.
C)II, III and IV only.
D)I, III and IV only.

A

Rationale
Solution: The correct answer is C.

Total salary reductions for qualified 401(k) and TSA is limited to $19,500 per year in 2020. Contributions to 401(k)s and 403(b)s are aggregated such that they may not exceed the total annual limit. The TSA has make-up provisions that allow certain employees to make up contributions that could have been made in the past but were not. All assets in qualified plans are part of the gross estate of the account owner. Employers may make matching contributions or contribute a fixed percentage of an employee’s compensation to a TSA.

211
Q

OASDHI earnings test

A

OASDHI: old age (or retirement), survivors’, disability, and health (or Medicare) benefits, which include hospital insurance and supplemental medical insurance. The program can be separated into two broad parts. The first part of OASDHI is the old-age, survivors’, and disability (OASD) insurance program known as Social Security. The second part of the OASDHI program is Medicare (HI).

The earnings test does not apply at, or after, normal age retirement. The monthly exempt amount is $4,050 ($48,600) in 2020 for those months in the year of normal retirement age BEFORE you actually reach normal retirement age. The test uses only earned income. No passive or portfolio income is used in calculating the earnings.

212
Q

cash balance pension plans

A

Cash balance plans are defined benefit plans due to the guaranteed investment returns and benefit formula, not simply a contribution amount. While cash balance plans provide guaranteed rates of return, they are not 100% guaranteed by the PBGC (PBGC has coverage limits). Cash balance plans use 3-year cliff vesting only.

213
Q
Hot Dog Moving Company (HDM) sponsors a 401(k) profit sharing plan. In the current year, HDM contributed 20% of each employee’s compensation to the profit sharing plan. The ADP of the 401(k) plan for the NHC is 2.5%. Alex, who is age 57, earns $177,778 and owns 7.5% of the company stock. What is the maximum amount that he may defer into the 401(k) plan for this year?
A)$8,000
B)$11,000
C)$14,500
D)$16,500
A

Rationale
The correct answer is C. Alex is highly compensated because he is more than a 5% owner, so the maximum that he can defer to satisfy the ADP Test requirements is 4.5% (2.5% + 2%) and because he is over 50, he can defer the additional $6,500 (2020) as a catch-up contribution. Alex can defer $8,000 (4.5% x $177,778) and $6,500 (the catch-up) for a total of $14,500.

214
Q

Which age can receive the social security retirement benefits?

A

The earliest age for social security retirement benefits is 62.

215
Q

defined benefit plan vs a SEP

A

defined benefit plan (expensive) or a SEP, which is extremely easy to set up and one that he can contribute around $11,000 to annually.
A SEP is much cheaper to set up and has no annual report requirements like the 401k.

He was be able to contribute as an employer to the SEP, which will be based on 20% of net income, around the $11,000 amount (without doing the full self-employment calculation).

216
Q

Roth IRA is subject to RMD?

A

You must take required minimum distributions (RMDs) from a traditional IRA starting at age 72. Unlike traditional IRAs, there are no RMDs for Roth IRAs during the account owner’s lifetime. Your account’s beneficiaries may need to take RMDs to avoid penalties.

The Roth 401(k) is subject to RMD

If an employee is still working in the company, then he will not subject to this company’s RMD

217
Q

What’s the eligibility of cash balance plan?

A

The standard eligibility rules are age 21 and one year of service, defined as 1,000 hours of service within a 12-month period

218
Q

Non qualified distributions from a designated Roth account associated with a 401k are subject to tax on a pro-rata basis.

A

Non qualified distributions from a designated Roth account associated with a 401k are subject to tax on a pro-rata basis. Her total account is the 15,000 invested and the 5,000 of income for a balance of $20,000. Since 75% (15,000/20,000) of the value in the account consists of basis and the remaining 25% consists of earnings (5,000/20,000), that same ratio of basis to income will apply to the $12,000 distribution. It is not a qualified distribution because she has not held the account for at least five years.

219
Q
Which of the following vesting schedules may a top-heavy qualified profit sharing plan use?
A)1 to 5 year graduated.
B)5-year cliff.
C)3 to 7 year graduated.
D)4 to 8 year graduated.
A

Rationale
The correct answer is A. As a result of the PPA 2006, qualified profit sharing plans must use a vesting schedule that provides participants with vested benefits at least as rapidly as either a 2 to 6 year graduated vesting schedule or a 3-year cliff vesting schedule. This requirement applies without regard to whether the profit sharing plan is a top-heavy plan. Options b, c, and d all vest less rapidly than the required schedule.

220
Q

A new comparability plan is a profit sharing plan.

A

The unit credit formula rewards many years of service. The flat percentage formula will work well, as long as the Doctor has ten years of service. The maximum benefits under IRC 415(b) are reduced for participation less than 10 years. The flat amount would provide higher benefits for Nurse Nancy compared to Dr. DJ on comparative basis.

221
Q

SEP-IRA plans are unique from defined contribution plans in which of the following areas:

I. Length of permissible exclusion from coverage based upon service.

II. Establishment date of the plan.

III. Income requirements for participation.

IV. Can be paired with another plan.

A)I, II and IV only.
B)I, II and III only.
D)None of the above.

A

Rationale
The correct answer is “B.” Employees can be excluded up to 3 years or age 21, whichever is longer. Plans can be established and funded up to the date of filing the entity tax return, including extensions. Employee needs to earn only $600 to be included in the plan.
C)I, II, III and IV.

Simplified Employee Pension Plan (SEP)
A SEP plan allows employers to contribute to traditional IRAs (SEP-IRAs) set up for employees. A business of any size, even self-employed, can establish a SEP.

222
Q

XYZ has a noncontributory qualified profit sharing plan with 300 employees in total, 200 who are nonexcludable (50 HC and 150 NHC). The plan covers 75 NHC and 35 HC. The NHC receive an average of 4% benefit and the HC receive 5.8%. Which of the following statements is (are) correct?

  1. The XYZ company plan meets the ratio percentage test.
  2. The XYZ company plan fails the average benefits test.
  3. The plan must and does meet the ADP test.

A)1 only.
B)2 only.
C)Both 1 and 2
D)1, 2 and 3.

A

Rationale
The correct answer is C. The plan does not have to meet the ADP test because it is a noncontributory plan. The ADP test would apply if there were a non safe harbor 401(k) plan. The plan meets the ratio percentage test and fails the average benefits test.

Ratio percentage test (75/150) divided by (35/50) equals 71.4% PASSES

Average benefits percentage test: 4% / 5.8% = 68.97% FAILS

223
Q

ADP Coverage Test:

A

This ratio must be at least 70% for the plan to pass.

Top-Heavy Test: compares the accumulated assets of key employees to non-key employees. Here the definition of a key employee differs from that of an HCEs, but the concept is similar. A plan is top-heavy if more than 60% of assets are attributable to key employees.

224
Q
Harry wants to retire at age 62 in the current year. To be eligible for reduced OASDHI retirement benefits, how many quarters of coverage must Harry have earned?
A)6
B)13
C)20
D)40
A

Rationale

The correct answer is “D.” Harry must have earned 40 quarters of coverage.

225
Q

IRA

A

In 2020, contributions are limited to the lesser of 100% of earned income or $6,000 or $7,000 if age 50 or over. Deductions may be taken even if an active participant, so being a non-participant is not a requirement. There are no age restrictions to make contributions into an IRA, just must have earned income (SECURE Act 2019). Contributions must be made prior to April 15 (or the mandated filing date for the year.) No extension to make the contribution is allowed after that date, even though an extension to file the return is granted.

226
Q

Which of the following plans is subject to PBGC coverage?
A)A cash balance plan for a local store selling nutrition supplements and drinks.
B)A target benefit plan for a publicly traded company.
C)An integrated defined benefit plan for an architect’s office that employees two owners, who are architects, three junior architects and one administrative assistant.
D)A new comparability plan with three groups for a firm with 26 employees.

A

Rationale
The correct answer is A. All DB plans (DB and cash balance) must be covered by PBGC except for professional firms with 25 or fewer employees. Choice b is not correct because it is defined contribution plan and is not subject to PBGC. Choice c is not correct because although it is defined benefit plan, it is not subject to PBGC because it is a professional firm with fewer than 25 employees. Choice d is not correct because it is defined contribution plan and is not subject to PBGC.

227
Q

Which of the following legal requirements apply to profit sharing plans?

I. Forfeitures must be used to reduce employer contributions or be reallocated to the remaining participant’s accounts.

II. Employer contributions must be allocated through a compensation-based formula.

III. Employer deductions for plan contributions are limited to 25% of the participants’ covered compensation.

IV. Allocations to a participant’s account cannot exceed the lesser of 100% of compensation or $57,000 in 2020.

A)I and II only.
B)II and III only.
C)II and IV only.
D)I, III and IV only.

A

Rationale
The correct answer is “D.” Statements “I”, “III” and “IV” are correct. Statement “II” is incorrect because there are other methods by which allocations can be made (i.e., age weighted, unit weighted, etc.).

228
Q

Because no tax deductions are taken for any premiums paid on the policy, the death benefits are tax-free

A

the employer is unable to deduct any contributions to a non-qualified plan until the employee actually takes constructive receipt

229
Q

maximum total annual additions to 401 (k) is $57,000

A

$19,500, the employee elective deferral

230
Q

Which of the following is NOT true concerning eligibility or “waiting periods”?
A)A qualified plan can require a waiting period of up to two years.
B)In order to have graded vesting, the eligibility period must be no longer than 12 months.
C)Any plan which has an eligibility period exceeding 12 months must have immediate vesting.
D)The eligibility period can vary between classes of employees.

A

Rationale
The correct answer is “D.” The eligibility requirements must be the same for all employees. All other statements are accurate.

231
Q

Which statement(s) is/are true for a target benefit plan?

I. It favors older participants.

II. It requires annual actuarial assumptions.

III. The maximum deductible employer contribution is 25% of covered compensation.

IV. The maximum individual allocation is the lesser of 100% of pay or $57,000 (2020).

A)I and IV only.
B)II and III only.
C)I, III and IV only.

D)I, II, III and IV only.

A

Rationale

The correct answer is “C.” Actuarial assumptions are needed at the inception of the plan to provide the target formula.

232
Q

A SEP is much cheaper to set up and has no annual report requirements like the 401k.

A

A rabbi trust is established to avoid constructive receipt.

233
Q

Employees do not self-direct their assets in a cash balance plan.

A

PBGC does not fully guarantee benefits.

234
Q

In an involuntary conversion, the date of realization, not the payment date, determines the date of recognition.

realized gains may, in some instances, be deferred or excluded such as the gain on sale of a personal residence.

A

realized gains on some personal use assets are recognized unless excluded.

realized losses on personal use assets are not recognized.

235
Q

deductible mileage to work: 0; The deduction is eliminated for years after 12/31/17

A

Punitive damages are always included in income. They may be excludible if they are the only recovery in a wrongful death case (The Alabama Rule).

236
Q

The gain on the sale of the home was $176,000 less $10,000 in selling expenses for an adjusted selling price of $166,000. The basis of the home sold was $132,000 for a realized gain on sale of $34,000. Under Section 121, a taxpayer who 1) owns and 2) lives in their primary residence for two years will be able to exempt up to $250,000 in gain if single ($500,000 if married filing joint.) Here the gain is below the $250,000, so all of the gain will be exempt.

A

The IRS provides guidelines for the amount of premiums that are deductible based upon the insured’s age. The amount of premiums paid is included in the medical expense deduction for total expenditures exceeding 7.5% of AGI (SECURE Act 2019) and is from AGI. The policy must be guaranteed renewable or non-cancelable to be qualified.

237
Q

Under Section 121, a taxpayer who 1) owns and 2) lives in their primary residence for two years will be able to exempt up to $250,000 in gain if single ($500,000 if married filing joint.) Here the gain is below the $250,000, so all of the gain will be exempt.

A

The IRS provides guidelines for the amount of premiums that are deductible based upon the insured’s age. The amount of premiums paid is included in the medical expense deduction for total expenditures exceeding 7.5% of AGI (SECURE Act 2019) and is from AGI. The policy must be guaranteed renewable or non-cancelable to be qualified.

238
Q

Only pension plans are required to have a joint and survivor annuity option. Profit sharing plans (including ESOPS) are NOT required to have a joint and survivor annuity option.

A

Cafeteria plans have a “use-it-or-lose-it” provision which requires any funds not used to pay qualified claims during the plan year be forfeited back to the plan sponsor. Forfeited funds cannot be rebated back to the individual employee who forfeited the funds.

a cafeteria plan must have only two benefits, one taxable (salary) and one pre-tax (FSA).

239
Q

Section 132 fringe benefits like health club dues, which must be provided “on the employer premises.” are taxable to employee.

A

Cafeteria plans have which of the following characteristics?

I. Must offer a choice between at least one qualified “pre-tax” benefit and one non-qualified “cash” benefit.

II. Medical Flexible Spending Accounts (FSAs) can reimburse medical expenses not covered by insurance for the participant and all dependents.

III. Changes in election amount during the plan year can only occur with a “qualifying change in family status.”

240
Q

Unrelated Business Taxable Income (UBTI):

A partnership interest in an investment enterprise, whether active or passive, is NOT subject to UBTI.

A

Direct investment in a business generates income which is UBTI.
Any investment which is purchased with “leverage” or borrowed funds generate UBTI except for a qualifying ESOP or LESOP.

241
Q

Vested options are taxable based on the value of the option to the extent the Fair Market Value exceeds the option price.

A

If a stock option is vested when it is received, and has a readily ascertainable value it is: Immediately taxable.

242
Q
Lisa, age 35, earns $175,000 per year. Her employer, Reviews Are Us, sponsors a qualified profit sharing 401(k) plan, which is not a Safe Harbor Plan, and allocates all plan forfeitures to remaining participants. If in the current year, Reviews Are Us makes a 20% contribution to all employees and allocates $5,000 of forfeitures to Lisa’s profit sharing plan account, what is the maximum Lisa can defer to the 401(k) plan in 2020 if the ADP of the non-highly employees is 2%?
A)$7,000
B)$13,500
C)$17,000
D)$19,500
A

Rationale
The correct answer is “A.” The maximum annual addition to qualified plan accounts is $57,000. If Reviews Are Us contributes $35,000 ($175,000 x 20%) to the profit sharing plan and Lisa receives $5,000 of forfeitures, she may only defer $17,000 ($57,000 - $35,000 - $5,000) before reaching the $57,000 limit (2020). However, she will also be limited by the ADP of the non-highly employees because she is highly compensated (compensation greater than $130,000). If the non-highly employees are deferring 2% then the highlys can defer 4% (2x2=4). Therefore, she is limited to a deferral of $7,000 ($175,000 x 4%).

243
Q

The employer under Profit-sharing plans is not required to make any particular percentage of profits. Though contributions must be substantial and recurring, the plan concerns itself more with allocation requirements rather than with contributions. As long as contributions are recurring, they need not be made in a year where the employer has not made a profit.

A

Roth monies are contributed with ‘already been taxed’ dollars, and therefore no longer taxable. And all Roth distributions are handled as follows: Withdrawal from a Roth IRA is treated as made first from direct contributions to the Roth IRA, then from conversion contributions (first-in first-out, or FIFO, basis), and then from earnings in the Roth IRA.

244
Q

A person will be able to defer $57,000 plus the catch up of $6,500 to the 401(k) plan, where she can only contribute $57,000 to the target benefit plan and the SEP

A

all life insurance in qualified plans is subject to income when purchased, regardless of the type.

245
Q

An individual can defer up to $19,500 (2020) plus an additional $6,500 catch up for all of their 401(k) and 403(b) plans combined.

Since he is 50 or older he can contribute the 19,500 + 6,500 = $26,000

A

Group survivor’s income insurance provides for the survivor of the employee. The beneficiary designation cannot be altered

246
Q

Once an individual has participated in a rollover where funds from IRA have been withdrawn and held and then reinvested, he or she is ineligible for such a transaction again for one year from the date of receipt of the amount withdrawn.

A

Employers generally contribute to Money Purchase Pension Plans, ESOPs, and Profit Sharing Plans. Employees contribute (thus contributory plans) to 401(k)s and Thrift Plans.

247
Q

Maximum contribution into a SEP is 25% or $20,000 per employee. The modification for the owner, a self-employed person is, would be: EE contribution % / 1 + EE contribution %. In this case the calculation would be .25 / 1.25 = .2; $100,000 x .2 = $20,000 maximum contribution for the business owner.

A

Only earned income qualifies for contributions into an IRA.

248
Q

Defined benefit plan should be taken as:

A)Base the benefit on the years of service and salary level of employees, while taking into consideration some of the benefits of Social Security.
B)Base the benefit retirees receive on a fixed percentage of every employee’s salary, limited to the annual compensation limit.
C)Define the benefit for retirees as a FIXED DOLLAR amount, regardless of income level.

A

The adoption of a cash balance plan is generally motivated by two factors: selecting a benefit design that employees can more easily understand than a traditional defined benefit plan, and as a plan that has more predictable costs associated with its funding.

Cash balance plans are defined benefit plans due to the guaranteed investment returns and benefit formula, not simply a contribution amount.

While cash balance plans provide guaranteed rates of return, they are not 100% guaranteed by the PBGC (PBGC has coverage limits).

Cash balance plans use 3-year cliff vesting only. Choice d is correct.

249
Q

The mandated COBRA requirement does not take effect until the employer has 20 or more employees.

A

Defined benefit plans favor older participants. It also requires PBGC coverage as mandated by law and needs to be actuarially calculated.
The maximum benefit listed is $230,000, but annual contributions cannot be made on an arbitrary basis.(任意依据。)

250
Q

The total of employer contributions, employee contributions and forfeitures allocated to a participant’s account cannot exceed the limits under Internal Revenue Code Section (IRC) 415(c).

IRC Section 415(d) provides for a cost of living adjustment to $56,000 in 2019, $57,000 in 2020, and $58,000 in 2021.

A

top-heavy key-employee maximum: $230,000

annual compensation limit: $285,000

employee contribution: $19,500

251
Q

the AGI limitation for a MFJ active participant ($104,000 - $124,000) (2020).
a Roth IRA : AGI limitation of $196,000 - $206,000 (2020).
Incidentally, 50 or older is allowed to make the $1,000 (2020) catch up contribution.

Since she is not an active participant but her husband is, she will utilize the spousal phase out limitation of $196,000 - $206,000 (2020) for traditional IRA.

A

Payments under a “golden parachute” are considered ordinary income. Additionally, any amounts under the Social Security cap will be subject to the OASDI tax. All amounts will be subject to Medicare tax. “Golden parachute” payments are also subject to an additional 20% excise tax. Because these are non-qualified plans, no lump sum treatment or IRA rollover options apply.

252
Q

A key employee is an individual who (1) owns more than 5% of the business, (2) is an officer with compensation greater than $185,000, or (3) owns greater than 1% of the business and has compensation greater than $150,000.

A

Tax-Sheltered Annuity (TSA) provisions:

The annual elective deferral limit may be increased by up to $3,000 for employees of certain organizations who have completed 15 years of service and meet certain other requirements.

because TSAs can only invest in mutual funds or annuities and not any direct investments.

the total excludable contributions must be for all prior years, not just the past three.

253
Q

The maximum service requirement that a profit sharing plan may impose as a condition of participation is 2 years

A

discretionary : 酌情决定的

254
Q

Profit sharing plan : requires that contributions be “substantial and recurring.” More specifically, an age-based profit sharing plan would be correct.

A

Profit sharing plan : A hybrid plan that uses a discretionary contribution but adjusts for age

255
Q

S Corporations and proprietorships cannot deduct any premiums for group health insurance for owners. Non-owner employee health premiums are fully deductible to both entities.

Premium costs of group health insurance paid by a partnership are passed through to the partner, who can deduct 100% of the costs on their individual tax return

A

Lodging as a fringe benefit is only excluded if
(1) it is furnished on the business premises (the place of work),
(2) it is furnished for the employER’s convenience,
and (3) the employee accepts it as a condition of employment.
A vacation stay does not meet any of the these criteria so it is fully taxable.

256
Q

Simple IRAs do not require the 20% withholding because they are not qualified plans.

Therefore, the entire account balance would have been distributed to him.

A

Participating in a 457 plan is not considered being an “Active Participant.” She can contribute and deduct her contribution to a Traditional IRA $6,000 (2020) since she is not an active participant and therefore not subject to an AGI limitation. She is unable to contribute to a Roth IRA because she is above the AGI limitation of $124,000 - $139,000 (2020). Because she is 50 or older she is allowed to make the $1,000 (2020) catch up contribution.

257
Q

a maximum IRA contribution of $6,000 (2020) or 100% of EARNED income ($3,500) since their joint AGI is below the Roth IRA phase-out threshold.

She has already contributed $1,000 to traditional IRA, so maximum available for the Roth is $2,500 ($3,500 earned less $1,000).

A

In a qualified retirement plan, the loan can be up to 50% of the employer’s accrued balance or $50,000, whichever is less. There are mandatory repayments which must occur at least quarterly.

The term of the loan can be for a maximum of five years, unless the proceeds are used to purchase a residence, then the loan can be for a longer period of time. Most plans have a minimum participation requirement of two years before loans are available. A reasonable interest rate must be charged. Many plans have a minimum loan amount of $10,000.

258
Q

SEPs prohibit forfeitures: Simplified Employee Pension plan (SEP)

A

IRC Section 457 plans are nonqualified deferred compensation plan, and therefore do not make the employee an “active” participant in a qualified retirement plan. The 401(k) is a qualified plan and the 403(b) and SEP are ‘wannabe’ be plans that would make the employee an “active” participant.

259
Q

Returns on portfolio assets and actuary funding are a concern in defined benefit plans.

A money purchase plan is defined contribution plan, NOT defined benefit plan. (remember the table in the book)

A

A person is not subject to 10% penalty from a profit sharing plan because this is a qualified plan and she is over 55.

Distributions from qualified plans are subject to mandatory 20% withholding.

260
Q

Involuntary termination of employment

A

非自愿终止雇佣

261
Q

As a result of the PPA 2006, qualified profit sharing plans must use a vesting schedule that provides participants with vested benefits at least as rapidly as either a 2 to 6 year graduated vesting schedule or a 3-year cliff vesting schedule. This requirement applies without regard to whether the profit sharing plan is a top-heavy plan.

A

ESOP distribution in stocks are NUA (Net Unrealized Appreciation). Employees in an ESOP may demand 25% of the current balance be diversified.

262
Q

The SIMPLE plan has a mandatory match from employer.

A

Short-term disability benefits usually start the eighth day of an illness (first day for an accident) and generally last no more than six months.

263
Q

the funding formula for the defined benefit pension plan will limit his defined benefit to $12,000 (5 x 120,000 x .02). Note that you would USE
2%
instead of the 1.5% because the plan is TOP heavy.

A

A top heavy plan mandates a 3-year cliff vesting or 2-6 graded vesting schedule (or faster if elected by sponsor.) Top heavy plans must make a minimum contribution of 3% to non-key employees. This PSP had already made a 5% contribution so no additional contribution is required. Top heavy plans are still qualified as long as minimum vesting and contribution requirements are met.

264
Q

70.5 and therefore is unable to make a contribution to a Traditional IRA.

A

SEP is an ACTIVE participant

265
Q

For 403 b, a person is not eligible for the long service catch-up because the museum is not a Health, Education, Religious (HER) organization.

The maximum contribution limits for 2020 are $19,500 plus the age 50+ catch-up of $6,500.

A

Tax Sheltered Annuity (TSA):

employers may make matching contributions or contribute a fixed percentage of an employee’s compensation to a TSA.

266
Q

Cash balance plans are defined benefit plans due to the guaranteed investment returns and benefit formula, not simply a contribution amount. While cash balance plans provide guaranteed rates of return, they are not 100% guaranteed by the PBGC (PBGC has coverage limits). Cash balance plans use 3-year cliff vesting only.

A

Qualified plans have tax-deferred contributions from the employee, and employers may deduct amounts they contribute to the plan.
Nonqualified plans use after-tax dollars to fund them, and in most cases employers cannot claim their contributions as a tax deduction.

267
Q

The defined benefit plan must cover the lesser of 50 people or 40% of all employees. In this case, the lesser would be 40% of 60, or 24 people.

A

Profit sharing plan: default contribution rate by employer is 3%. The fact that the plan is top heavy is irrelevant since all employees are receiving a contribution greater than 3%..

268
Q

Which statement(s) is/are true regarding qualified profit-sharing plans?

I. A company must show a profit in order to make a contribution for a given year.

II. A profit-sharing plan is a type of retirement plan and thus is subject to minimum funding standards.

III. Forfeitures in profit-sharing plans must be credited against future years’ contributions.

IV. Profit-sharing plans should make contributions that are “substantial and recurring.”

A)IV only.
B)I and III only.
C)I, II and IV only.
D)I, II, III, and IV.

A

Rationale
The correct answer is “A.” Profits are not required to make contributions to a profit-sharing plan. Minimum funding is required only in pension plans. Forfeitures may be reallocated to remaining employee accounts.

269
Q

Defined benefit plan: default top heavy rate is 2%.

Note that you would use 2% instead of the 1.5% because the plan is top heavy

A

Roth IRA early withdraw:

Earnings will be TAXABLE, but not for basis and conversion.

Contributions are NOT subject to the early withdraw penalty 10%. Other parts, such as earnings and conversion could pay penalty if the number of holding period on the account is less than 5 years.

Roth distributions are tax free if they are made after 5 years and because of 1)Death, 2)Disability, 3) 59.5 years of age, and 4)First time home purchase.

Withdrawal from a Roth IRA is treated as made first from direct contributions to the Roth IRA, then from conversion contributions (first-in first-out, or FIFO, basis), and then from earnings in the Roth IRA.

270
Q

Defined contribution plan has:

I. Individual accounts.

III. Retirement benefits based on account values.

A

Defined benefit plan has:

II. Actuarial assumptions required.

IV. Employer bears investment risk.

271
Q

Flexible saving accounts:

FSAs are funded with pre-tax dollars as part of a cafeteria plan.

Unused funds at the end of the year revert to the employer who is prohibited from returning the funds directly to the forfeiting employee.

Dependent care FSAs are limited to $5,000 per year.

Medical FSA funds can only pay for medical expenses. Payment of child care expenses would be non-qualified expenses.

A

SIMPLE IRAs require a 25% penalty for early withdraws in the first two years if the participant does not meet any of the early withdrawal exceptions.

272
Q

FSAs are funded with pre-tax dollars as part of a cafeteria plan. Unused funds at the end of the year revert to the employer who is prohibited from returning the funds directly to the forfeiting employee. Dependent care FSAs are limited to $5,000 per year. Medical FSA funds can only pay for medical expenses. Payment of child care expenses would be non-qualified expenses.
C)

A

An excess benefit plan extends the same benefits to employees whose contributions to the plan are limited by Section 415 (e.g., employee earns $285,000 yet receives $57,000 contribution instead of $70,000 contribution due to Section 415 limitation on a 25% money purchase plan). An excess benefit plan would put additional $13,000 into non-qualified retirement plan. Do not confuse with a SERP which provides benefits in excess of the Section 415 limits AND ignores the covered compensation limits (i.e., $285,000 in 2020) applied to qualified plans.